Схема перекрестного выключателя с трех мест: Страница не найдена — Аква-Ремонт

Содержание

Схемы Подключения Проходных И Перекрестных Выключателей

Схема управления освещения с трех мест Схема подключения двухклавишных устройств состоит их двух параллельных схем, по одной для каждой лампы.


Если есть желание управлять работой двух ламп из трех или четырех точек, то придется приобрести по два перекрестных переключателя.

При замыкании концов щупов, прибор издает звуковой сигнал, что весьма удобно, так как нет необходимости смотреть на дисплей прибора. Что касается дешевых китайских выключателей, то в основном, подобной схемы нет, поэтому приходится концы вызванивать прибором.
Подключение проходного выключателя управление освещением из 3 х мест

Несмотря на удобство и простоту реализации, такое соединение довольно редко используется современными электриками — считается, что несколько проходных моделей работают более надежно.

Проходные выключатели обеспечивают управление одним источником света из двух разных точек, имеют три контакта 1 входной, 2 выходных. К двухклавишным прокладывается 5 к первому проходному, 8 к промежуточным и 6 ко второму проходному.


Из скольких мест можно включать свет?

В поворотных контакты замыкаются при помощи поворотного механизма.

Такая конструкция позволяет одновременно отключать или включать по два контакта. Схема перекрестного выключателя.

Как подключить перекрёстный выключатель шнайдер, Schneider, ремонт

Функции перекрестного переключателя

Визуально отличить их никак нельзя, но по функционалу данные выключатели сильно отличаются. Она показывает, как организовать в помещении стабильную работу автономного управления освещением из разных точек.


Для возможности управлять светильниками с трех мест, между ПВ устанавливают промежуточный переключатель, роль которого выполняет перекрёстный выключатель.

Схема управления из двух мест Схема управления светом из трех мест, где 1 — проходной выключатель; 2 — перекрестный выключатель; 3, 5 — подрозетники для проходных выключателей; 4 — подрозетник для перекрестного выключателя; 6 — ответвительная коробка; 7 — к корпусу светильника Схема управления из трех мест Обозначение 6 видов перекр-х вык-лей На схемах каждый вид электроприборов имеет своё обозначение. Накладной или наружный переключатель устанавливают на стену без предварительной подготовки выемки или монтажа базового блока.

Решив сделать у себя дома систему переключателей в которой управление освещением осуществляется из трех и более мест, необходимо включать в схему перекрестные переключатели, располагая их между двумя проходными.

Ответы на 5 часто задаваемых вопросов Есть вопросы, которые задают люди при выборе таких систем: Сколько проводов необходимо подводить переключателям?

В схеме включения освещения, задействуются два и более, подобных выключателей. Выключатель работает в двух положениях и имеет два контакта, которые замыкаются и размыкаются одновременно.

Как отличить перекрестный переключатель от проходного? Ответы на 5 часто задаваемых вопросов Есть вопросы, которые задают люди при выборе таких систем: Сколько проводов необходимо подводить переключателям?
Схема проходного выключателя из трех и более мест

Обычные выключатели

Как отличить перекрестный переключатель от проходного? Компания выпускает оборудование для электротехники и энергетического машиностроения.

Схема подключения перекрёстного выключателя ABB не отличается от схем подключения аппаратуры других фирм. Вариант разводки проводов для подключения двухклавишных проходных выключателей.

Чтобы не ошибиться при их коммутации, давайте рассмотрим схему подключения перекрестного переключателя.


Как все три выключателя соединить видно на рисунке. При нажатии на кнопку любого из трех выключателей происходит размыкание цепочки. Таким образом, получаем систему освещения, независимо управляемую из разных точек.

Способ монтажа перекрестных выключателей Подключение перекрестного выключателя зависит от модели: Встроенный. Схема подключения двух проходных выключателей Подобная схема может оказать существенную помощь в организации освещения на лестнице в двухэтажном доме , в длинном коридоре или в проходной комнате. Прежде чем понять, для чего применяется перекрёстный выключатель, нужно разобраться, как работает проходной выключатель. ABB Basic 55 белый:.

Как подключить проходной выключатель — схема управления светильником из 3 мест


Перекрестные модели применяются только в комплекте с проходными, но, при этом их обозначение на схемах идентично. Чтобы не ошибиться при их коммутации, давайте рассмотрим схему подключения перекрестного переключателя. Схема перекрестного выключателя. Итак, подведем итоги: 1.

Все соединения производятся в распределительной коробке, и в нашем случае получилось семь соединений скруток. При желании можно по схеме собрать своими руками, если есть подходящие комплектующие. Система для управления светом из нескольких мест устроена иначе. Но после его установки в разветвительную коробку рабочий провод отводится не на лампу, а на контакты выключателя в данном случае, это один из проходных ; Через распределительную коробку производится последовательное соединение контактов.

На любое, до В. Для возможности управлять светильниками с трех мест, между ПВ устанавливают промежуточный переключатель, роль которого выполняет перекрёстный выключатель. Переделка 2-х клавишного обычного выключателя в проходной Как подключить перек-ый перек-ль — 2 способа прокладки проводов После составления схемы её необходимо смонтировать и подключить.
Как подключить перекрестный промежуточный выключатель переключатель с трех мест схема перекрестного

Работа схемы с 3 видами вык-лей – обычным, проходным и перекрёстным

Как видите, при любой комбинации положения контактов выключателей мы всегда сможем включить и выключить свет с любого из них.

Если же, у вас все равно остались вопросы, по схеме подключения перекрестного переключателя, обязательно оставляете их в комментариях к статье, постараемся помочь!

Два выхода одного выключателя соединяются с двумя выходами второго выключателя, а два других выхода этого выключателя подсоединяются к двум другим выходам первого выключателя. Ответы на 5 часто задаваемых вопросов Есть вопросы, которые задают люди при выборе таких систем: Сколько проводов необходимо подводить переключателям? Вставляются такие приборы в разрыв двух проводов, соединяющих проходные переключатели.

Называется прибор так потому, что при переключении он меняет местами подключение подходящих проводов к отходящим — крест накрест. Перекрестный выключатель работает только в комплекте с проходными выключателями и в схемах освещения включается между ними. Фото — схема работы перекрестного выключателя Главным отличием проходных выключателей от перекрестных является то, что первые могут использоваться самостоятельно, а вторые — нет. К каким клеммам подключаются провода.

Статья по теме: Энергосбережения на сто

Навигация по записям

Все детали изготавливаются из качественных материалов и проходят контроль на всех этапах производства. Принцип функционирования Ниже представлена схема подключения промежуточных выключателей, обеспечивающая независимое включение и выключение света из двух разных мест. Он также имеет два контакта, такой же механизм переключения контактов, но отличается способом их переключения. Устройства, отключающие свет, не исключение.

Чтобы не ошибиться, что за выключатель, следует ознакомиться со схемой включения, которая присутствует на корпусе выключателя. Для управления одним прибором, излучающим свет, из трех и более мест, применяют перекрёстные выключатели.

Проходные выключатели

При нажатии на кнопку любого из трех выключателей происходит размыкание цепочки. В распределительную коробку от щитка протягивается нулевой провод.

Нажав на клавишу выключателя ПВ2, цепь замкнётся. Как видите, при любой комбинации положения контактов выключателей мы всегда сможем включить и выключить свет с любого из них.

Двойной перекрестный выключатель Schneider Electric

Схема подключения перекрестного переключателя – RozetkaOnline.COM

Перекрестный или промежуточный переключатель применяется в системах, где управление освещением осуществляется из трех, четырех и более мест. При этом, обязательно два переключателя в системе – проходные (схему подключения которых мы уже подробно рассматривали в статье “Схема подключения проходного выключателя (переключателя)“), а вот все остальные это перекрестные. Именно они обеспечивают возможность практической реализации таких схем, когда управлять одной люстрой можно из трех, четырех, пяти и даже из ста различных мест.

 

 

Довольно часто, приходя в магазин электрооборудования с желанием купить переключатели часть из которых проходные, а часть перекрестные и не получив квалифицированной помощи, наши читатели сталкиваются с проблемой, как отличить между собой перекрестный переключатель и проходной?

Действительно, внешне они идентичны, более того, точно так же выглядит обычный одноклавишный выключатель, поэтому ошибиться очень легко.

Основная отличительная особенность перекрестного переключателя, на которую и нужно обращать внимание при покупке, связана со схемой его подключения – это количество подсоединяемых проводов и соответственно клемм для них на механизме.

 

Запомните, для работы одноклавишного перекрестного переключателя требуется ЧЕТЫРЕ провода, для проходного переключателя – ТРИ, а обычного выключателя ДВА. В случае с двухклавишными устройствами (и да, двухклавишные перекрестные переключатели так же встречаются), количество подключаемых проводов соответственно увеличивается вдвое, для каждого случая.

Обычно, на обратной стороне перекрестного переключателя, рядом с клеммами для подключения проводов, нанесены следующие обозначения:

 

В данном случае, в качестве примера, использован перекрестный переключатель ABB Busch-Jaeger серии Basic55. Как видите, у него четыре пружинных зажима для подключения проводов. Чтобы не ошибиться при их коммутации, давайте рассмотрим схему подключения перекрестного переключателя.

 

На схеме указаны также два проходных переключателя, без них тяжело разобраться в принципе работы перекрестного выключателя, так как он используется обычно как минимум третьим в схеме и без двух проходных не применяется.

По большому счету, перекрестный переключатель является связующим компонентом между проходными, поэтому, как видно на схеме, в него приходят две жилы с первого проходного выключателя и выходят две на второй.

Теперь, я думаю, вам становятся понятны обозначения, нанесенные на его тыльной стороне, рассмотренные в начале статьи. Две стрелки, направленные внутрь промежуточного выключателя (две верхние) показывают клеммы для пары проводников идущие с первого проходного переключателя

 

а к клеммам со стрелками, указывающими наружу, подключаются провода, идущие на второй, конечный проходной переключатель. 

Соответственно подключение проводов к перекрестному переключателю необходимо выполнять именно в этом порядке – две жилы, идущие от одного проходного переключателя в схеме в одну пару клемм, а две другие жилы, идущие ко второму проходному переключателю в схеме, ко второй паре клемм.

 

 

Таким образом, промежуточный переключатель имеет два основных режима работы.

Первый: Когда сигнал, идущий между проходными переключателями не изменяется. Можно считать, что провода неразрывны, это равносильно схеме просто из двух проходных переключателей. Условно это выглядит так:

Второй: Когда сигнал перенаправляется, провода, идущие к конечному проходному переключателю, меняются местами между собой, иными словами перекрещиваются с проводниками приходящими от первого проходного переключателя. Условно это выглядит так:

Эти изображения наглядно иллюстрируют схему работы перекрестного выключателя, теперь, я думаю понятно, как он действует и почему он так называется.

 

Подробная пошаговая инструкция по подключению и установке перекрестного переключателя описана ЗДЕСЬ.

 

Итак, подведем итоги:

1. Решив сделать у себя дома систему переключателей в которой управление освещением осуществляется из трех и более мест, необходимо включать в схему перекрестные переключатели, располагая их между двумя проходными.

 

2. Для правильной работы к промежуточному переключателю должны быть проложены по две жилы от каждого из проходных переключателей, в общей сложности четыре провода.

Если в системе два перекрёстных выключателя (а это соответственно четыре места управления светом), то они соединяются по тому же принципу, последовательно: от первого проходного выключателя две жилы к первому перекрестному, дальше две жилы ко второму перекрестному, а уж от него две к конечному проходному.

Подробнее схема управления светом, состоящая из четырех переключателей представлена на изображении ниже.

 

3. Подключение проводов к перекрестному переключателю осуществляется в следующем порядке: две жилы от первого проходного переключателя в первую пару клемм, а два провода, по схеме выходящих дальше (неважно на следующий перекрестный переключатель или на последний проходной), подключается ко второй паре клемм.

4. Чтобы при покупке отличить перекрестный переключатель от проходного, необходимо смотреть на схему подключения указанную обычно с тыльной стороны, а также на количество подключаемых к устройству проводов и клемм для них, у перекрестного выключателя их ЧЕТЫРЕ, а у проходного ТРИ.

Это основная информация, которую необходимо знать, чтобы правильно подсоединить перекрестный переключатель, схема подключения которого, оказывается не такая уж и сложная, если разобраться в принципе его работы.

Если же, у вас все равно остались вопросы, по схеме подключения перекрестного переключателя, обязательно оставляете их в комментариях к статье, постараемся помочь!

Схема подключения проходного выключателя с 3х мест

Традиционная схема подключения проходного выключателя с 3х мест не сложна в выполнении, но предполагает обязательное соблюдение правил самостоятельного подсоединения.

Такой вариант подключения является оптимальным при наличии определенной конфигурации помещений.

Подключение проходного выключателя из 3 х мест

Проходного типа выключатели представляют собой удобные и функциональные переключатели, которые в процессе воздействия на клавишу, способны выполнить переброс главного контакта между двумя другими.

Принципиальным отличием подключения переключателя проходного типа от установки классических двухполюсных устройств являются следующие параметры:
  • последовательное подключение коммутаторов друг к другу;
  • замена процесса размыкания переключением фазы;
  • входных контактов в два раза меньше, чем выходных;
  • парные полюса на коммутаторах в обязательном порядке должны «смотреть» друг на друга.

Электромонтажные работы, связанные с самостоятельной установкой выключателя проходного типа с трёх мест, характерно соблюдением схемы, представленной соединительной коробкой, лампочками, а также выключателями и проводами.

В качестве источника освещения могут использоваться светильники с традиционными лампочками накаливания, энергосберегающие или светодиодные приборы.

Проходные переключатели, также известные, как перекидные, дублирующие и лестничные выключатели, по внешнему виду не имеют существенных отличий от обычных изделий, но позволяют управлять освещением с разных мест.

Где применяется система трех выключателей?

Обустройство прибора, позволяющего выполнять управление различного типа светильниками в трех разных точках, позволяет обеспечить практичность и экономичность эксплуатации системы внутридомового и уличного освещения. Это оптимальный вариант в многоэтажных частных домовладениях.

Рациональным является использование такой системы электрической проводки для освещения придомовой территории или приусадебного участка в условиях загородного домовладения.

Схема соединения выключателей между собой: посередине — крестовой с 4 контактами для соединения остальных переключателей

Также подобный вариант часто применяется в помещениях с несколькими спальными местами, что позволяет управлять освещением, не вставая с кровати.

Допускается использование такой системы в качестве лестничной подсветки и для освещения подъездов.

Применение схемы подключения выключателя проходного типа с трёх мест является наиболее удобным, практичным и экономным при обустройстве системы освещения.

Принцип перекрестного отсоединителя

Перекрестного типа выключатели очень похожи внешним видом на традиционное и популярное одноклавишное устройство, а основная разница состоит в наличии внутри корпуса четырех клемм. Название «перекрестный» обусловлено двумя электрическими линиями, которые переключаются.

Перекрестные отсоединители способствуют одновременному разъединению первого и второго выключателя, после чего осуществляется их синхронное соединение. Именно перемещением контактов объясняется зажигание и выключение источников света.

Подключение проходного выключателя схема с трех мест

Количество точек варьирует, но при большом их количестве значительно осложняется коммутация всех элементов внутри распределительной коробки.

Особого внимания требует грамотное подсоединение концов электрических кабелей, что гарантирует бесперебойную работоспособность и безопасность эксплуатации всей системы.

Элементы и составные части схемы подключения

Прежде чем приступить к самостоятельному подключению прибора управления освещением с трёх мест, необходимо приобрести основные расходные материалы, представленные:

  • соединительной коробкой;
  • светильниками с обычными лампами накаливания, светодиодными или энергосберегающими осветительными приборами;
  • парой переключателей проходного типа;
  • переключателем перекрестного типа;
  • электрическими проводами.

Инструкция по подключению тройного выключателя

Выключатели, которые могут быть задействованы в обустройстве системы — перекидные, дублирующие или лестничные, удобнее и практичнее традиционных устройств, но несколько сложнее в монтаже своими руками. Допускается установка одно- или двухклавишных устройств.

Первый вариант имеет три контакта. Кроме всего прочего, в процессе монтажа потребуется использовать набор отверток и гаечных ключей, монтажный нож и пассатижи, а также бокорезы.

При необходимости выполнить внутреннюю проводку требуется подготовить перфоратор и дрель с алмазным кругом, а для внешней установки используются традиционные кабель-каналы или гофрированная труба.

Тройной проходной выключатель – схема подключения

Стандартная схема включения, согласно которой монтируются устройства в трех точках, незначительно отличается от двухточечной установки.

Перекрестный переключатель имеет в схеме следующую функциональную нагрузку:

  • транзисторный аппарат, не взаимодействующий с парой других включателей осветительных приборов;
  • самостоятельный аппарат, размыкающий схему и обеспечивающий работоспособность части осветительных приборов.

Если выключатель проходного типа, устанавливаемый для пары точек, предполагает применение трёхжильного электрического кабеля, то для обустройства третьей точки используется пять контактов.

В этом случае пара контактов подсоединяется к одному из маршевых выключателей, а еще пара — для подведения ко второму устройству. Свободный прибор используется в качестве транзитного устройства.

Важно помнить, что присутствующий в схеме подключения транзитный контакт является обязательным, так как используется для включения в электрическую цепь и обеспечения работоспособности третьей точки подключения.

Монтаж

Процесс самостоятельного подключения выполняется в соответствии со следующими рекомендациями:

  • определение на проходном устройстве места расположения общей клеммы;
  • подведение к первому выключателю, установленному рядом с распределительной коробкой, «фазы» и последующая фиксация на общей клемме при помощи оранжевого или красного провода;
  • подсоединение к выходным клеммам внутри проходного выключателя пары оставшихся свободных проводов;
  • подведение ко второму выключателю кабеля и последующая фиксация его в соответствии с цветовой маркировкой;
  • подсоединение внутри распределительной коробки оранжевого или красного провода от второго переключателя к «фазе» осветительного прибора;
  • подсоединение внутри распределительной коробки двух свободных проводов к кабельной жиле от первого переключателя в соответствии с цветовой маркировкой.

Монтаж трехклавишного выключателя

На заключительном этапе необходимо выполнить подсоединение кабельной жилы «ноль» и «земля» внутри распределительной коробки к однотипному по назначению проводу, который затем заводится в осветительный прибор.

После того, как будет полностью выполнено соединение, необходимо тщательно подтянуть все скрутки, при необходимости выполнить лужение, а также заизолировать оголенные участки кабеля.

В быту используются не только понижающие, но и повышающие трансформаторы. Как из 12 вольт сделать 220 и в каких случаях это может быть необходимо, читайте внимательно.

О том, как проверить работоспособность конденсатора при помощи мультиметра, читайте на этой странице и следуйте инструкции.

Необходимость установки выключателей проходного типа обуславливается особенностями помещения, в котором установлены светильники, требующие регулирования работы из разных точек.

Грамотная установка обеспечивает удобство и комфорт эксплуатации, а при необходимости есть возможность усовершенствовать систему и задействовать даже большее количество точек.

Видео на тему

Схема подключения проходного выключателя с 3х мест

Зачем нужны проходные выключатели

Включение света в длинном темном коридоре может быть довольно неудобным, если есть лишь один выключатель, расположенный в конце комнаты. Наиболее рациональна установка проходных переключателей (другое название — перекрестные выключатели) в разных сторонах комнаты.

Так можно будет включить, выключить свет сразу после входа в коридор. Это особенно актуально в подъезде дома, где квартиры расположены одной линией по длинной лестничной площадке, на лестничных пролетах, в офисах, производственных помещениях.

Еще один вариант использования такой схемы управления — большая спальня с несколькими кроватями. Если установить проходные переключатели у каждого спального места, можно включить лампочку, не вставая. Монтирование таких устройств оправдано на дачах, приусадебных участках, дворах частных домов. Включать свет можно на выходе из дома — после завершения дел нет необходимости идти в темноте.

Типы выключателя на 3 точки


Выключатели с трех мест представлены двумя типа изделий: проходными и перекрестными. Последние не могут использоваться без первых. По принципу работы перекрестные делятся на:

  1. Клавишные.
  2. Поворотные. Для замыкания контактов используется поворотный механизм. Представлены разнообразным дизайном и обойдутся дороже обычных.

С учетом монтажа перекрестные делятся на:

  1. Накладные. Монтаж производится поверх стены, не требует создания в стене выемки для установки блока. Если отделка помещения не запланирована, то такой вариант идеален. Вот только такие модели недостаточно надежные, ведь подвержены внешним факторам;
  2. Встроенные. Устанавливаются в стену, подходят для работ по разведению проводки во всех типах зданий. Предварительно готовится отверстие в стене по размерам коробки переключателя.

Проходной

Проходной выключатель схема

В проходном выключателе в отличие от классической модели встроено три контакта и механизм, который объединяет их работу. Главное преимущество изделия – возможность проводить включение или выключение с двух, трёх или более точек. Второе наименование такого выключателя «перекидной» или «дублирующий».

Конструкция проходного выключателя с двумя клавишами напоминает два независимых друг от друга одноклавишных выключателя, но с шестью контактами. Внешне проходной от обычного выключателя не отличить, если бы не специальное обозначение на нем.

Схема подключение проводов проходного выключателя в распредкоробке


Схема без заземляющего проводника. Теперь самое главное это правильно собрать схему в распределительной коробке. В нее должны заходить четыре 3-х жильных кабеля:

  • кабель питания с автомата освещения распредщитка
  • кабель на переключатель №1
  • кабель на переключатель №2
  • кабель на светильник или люстру

При подключении проводов удобнее всего ориентировать по цвету. Если будете использовать трехжильный кабель ВВГ, то у него наиболее распространены две цветовые маркировки:

  • белый(серый) – фаза
  • желто зеленый – земля

или второй вариант:

Чтобы подобрать более правильную фазировку во втором случае, ориентируйтесь на советы из статьи “Цветовая маркировка проводов. ГОСТы и правила.”

  1. Сборка начинается с нулевых проводников. Соединяете нулевую жилу с кабеля вводного автомата и ноль отходящий на светильник в одну точку посредством клемм ваго.
  2. Далее нужно соединить все жилы заземления, если у вас есть заземляющий проводник. Аналогично нулевым проводам “землю” с вводного кабеля объединяете с “землей” отходящего кабеля на освещение. Этот провод подключается к корпусу светильника.
  3. Осталось правильно и без ошибок подключить фазные проводники. Фазу с вводного кабеля нужно соединить с фазой уходящего провода на общую клемму проходного выключателя №1. А общий провод с проходного выключателя №2 отдельным зажимом wago соединить с фазной жилой кабеля на освещение. Выполнив все эти подключения остается лишь соединить между собой второстепенные (отходящие) жилы с выключателя №1 и №2 между собой. Причем абсолютно не важно как вы их соедините.

Можно даже перепутать цвета. Но лучше все же придерживаться расцветки, чтобы не запутаться в будущем. На этом можно считать схему полностью собранной, подавать напряжение и проверять освещение.

Основные правила подключения в этой схеме которые вам нужно запомнить:

  • Фаза с автомата должна приходить на общий проводник первого выключателя
  • Эта же фаза должна выйти с общего проводника второго выключателя на лампочку
  • Два остальных вспомогательных проводника, соединяются между собой в распредкоробке
  • Ноль и земля подаются напрямую без выключателей сразу на лампочки

Перекрестный

Перекрестные модели с 4 контактами, что позволяет одновременно подключить два контакта. В отличие от проходных, перекрестные модели не могут использоваться самостоятельно. Их устанавливают в комплекте с проходными, на схемах обозначают идентично.

Напоминают такие модели два спаянных одноклавишных выключателя. Специальными металлическими перемычками соединены контакты. Всего одна кнопка выключателя отвечает за работу системы контактов. При необходимости перекрестную модель можно сделать самому.

Принцип работы перекрестного отсоединителя

Проходной прибор включения и выключения света внутри имеет четыре клеммы — на вид такой же, как обычные выключатели. Такое внутреннее устройство необходимо для крестообразного соединения двух линий, которые будет регулировать выключатель. Отсоединитель в один момент может сделать расключение двух оставшихся выключателей, после чего их вместе соединяет. Результатом становится включение-выключение света.

Для создания схемы применяют два и более проходных выключателя. Схема может включать любое количество проходных устройств, но увеличение их числа будет серьезно усложнять работу — необходимо четко знать порядок расположения кабелей и соединений в коробке.

Конструкция и особенности проходных выключателей

Внешней проходной прибор не отличается от стандартного. Разницу можно заметить только при осмотре изделия снизу – производители наносят на корпус треугольники, направленные по горизонтали вниз. Второй вариант различия – 3 клеммы с медными контактами. Одна располагается сверху, а две – снизу. Также проходное устройство коммутируется через трехжильный кабель ВВГ-нг или NYM с сечением 1,5 мм².

В зависимости от количества кнопок существуют двухклавишные, одноклавишные и трехклавишные модификации.


Разница между проходным и обычным выключателем.

В сравнении с классическими двухполюсными моделями подключить проходник нужно по следующему принципу:

  • последовательное подсоединение коммутаторов;
  • фаза не размыкается, а переключается на вторую линию;
  • контактов выхода больше, чем контактов входа.

Парные полюса коммутаторов располагаются друг против друга.

Нюансы выбора

Перед покупкой проходного выключателя требуется учитывать:

  • Способ крепления – зависит от типа проводки. Накладные устанавливаются на поверхность при помощи дюбель-саморезов. Встроенные – в подрозетниках на ножках-распорках.
  • Степень защиты – для спальни или коридора подойдут модели с IP03, для ванной – с IP04- IP05, для улицы – с IP55.
  • Тип контактных зажимов. Винтовые с прижимными пластинами отличаются надежностью. Безвинтовые пружинные проще в плане монтажа.
  • Маркировка клемм – используются обозначения N (ноль), L (фаза) и земля (заземление). Буквами I и O маркируется положение кнопок при включении и выключении.

По типу управления проходники бывают клавишными, сенсорными, с ПДУ.

Подключение проходного переключателя

В первую очередь необходимо правильно подключить сам выключатель в подрозетнике. Снимаете клавишу и накладные рамки.

В разобранном состоянии можно легко увидеть три контактных клеммы.

Самое главное – это найти общую из них. На качественных изделиях с обратной стороны должна быть нарисована схема. Если вы в них разбираетесь, то можно легко сориентироваться по ней.

Если же у вас бюджетная модель, или для вас любые электрические схемы темный лес, то на помощь придет обыкновенный китайский тестер в режиме прозвонки цепи, или индикаторная отвертка с батарейкой.

При помощи щупов тестера попеременно касаетесь всех контактов и ищете тот, на котором тестер будет “пищать” или показывать “0” при любом положении клавиши ВКЛ или ВЫКЛ. Еще проще это сделать индикаторной отверткой.

После того как вы нашли общую клемму, на нее нужно подключить фазу с кабеля питания. На остальные клеммы присоединяете два оставшихся провода.

Причем какой из них куда, не имеет существенной разницы. Выключатель собирается и закрепляется в подрозетнике.

Со вторым выключателем проделываете ту же самую операцию:

  • ищите общую клемму
  • подключаете на нее фазный проводник, который будет идти на лампочку
  • на оставшиеся подсоединяете две другие жилы

Перекидные выключатели — схема управления освещением из 3-х мест


А что делать, если вы хотите управлять одним освещением из трех точек и более. То есть выключателей в цепи будет 3, 4 и т.д. Казалось бы нужно взять еще один проходной выключатель и все.

Однако выключатель с тремя клеммами здесь уже не подойдет. Так как соединяемых проводов в распредкоробке будет четыре.

Здесь вам на помощь придет перекидной, или как его еще называют крестовой, перекрестный, промежуточный выключатель. Его ключевое отличие состоит в том, что он имеет четыре выхода – два снизу и два сверху.

И устанавливается он как раз таки в промежутке между двумя проходными. Находите в распаечной коробке два второстепенных (не основных) провода от первого и второго проходного выключателя.

Рассоединяете их, и подключаете между ними перекидной. Те провода что приходят с первого подключаете – на вход (ориентируйтесь по стрелочкам), а те что уходят на второй – к выходным клеммам.

Всегда проверяйте схему на выключателях! Зачастую бывает, что вход и выход у них находится на одной стороне (верх и низ). Например схема подключения перекидного Legrand Valena: 

Естественно сам перекидной запихивать в распаечную коробку не нужно. Достаточно завести туда концы 4-х жильного кабеля от него. А сам выключатель тем временем располагаете в любом удобном месте – возле кровати, в середине длинного коридора и т.д. Свет вы сможете включать и выключать из любой точки.

Самое главное преимущество этой схемы в том, что ее можно изменять до бесконечности и добавлять сколько угодно перекидных выключателей. То есть проходных будет всегда два (в начале и конце), а в промежутке между ними 4, 5 или хоть 10 перекидных.

Применение схемы

В длинных коридорах часто применяют проходные выключатели. Проходные модели предназначаются для удобства подачи или удаления напряжения осветительных приборов из разных концов помещения. Чаще всего такую схему применяют:

  • В длинном коридоре с выходами из разных комнат. На выходе ставят один переключатель, посередине – второй, в конце – третий.
  • Во дворах загородных и частных домов, дач. Коммутационные устройства монтируются на выходе из дома и хозяйственных постройках.
  • В многоквартирных трехэтажках. На первом этаже ставится прибор, чтобы выполнить включение. Отключить освещение можно на втором и третьем этажах.
  • В детской комнате с несколькими кроватями. Решение предусматривает один прибор на входе и два рядом со спальными местами.
  • Лестничные пролеты и площадки коттеджей за городом. Один девайс монтируется на начале лестницы, выключение осуществляется со второго этажа или около чердака.

При помощи проходного выключателя обеспечивается экономия электроэнергии.

Недостатки проходных выключателей

Одним из минусов проходных выключателей является необходимость штробления стен под них. Организация электролинии с проходным выключателем имеет несколько минусов:

  • затраты времени и сил на штробление стены при скрытой проводке;
  • необходимость подключения пульта ДУ через обычную клавишу для моделей с датчиками движения;
  • нерентабельность для квартиры из-за штроб под кабель и установки дин-рейки;
  • сложности с определением клемм;
  • отсутствие четкости положений «ВКЛ» и «ВЫКЛ».

Эксперты отмечают, что устройства больше подходят для загородных коттеджей, дач, частного сектора, чем для квартиры.

Проходной выключатель удобно использовать в помещениях с несколькими группами осветительных приборов. Устройство обеспечивает комфорт управления светом, безопасность перемещения людей. В настоящее время схема проходного переключателя света, управляемая с трех мест, не привязывается к планировке жилого объекта.

Ошибки подключения

Многие на этапе поиска и подключения общей клеммы в проходном выключателе совершают ошибку. Не проверяя схему, наивно считают, что общая клемма это та, где всего один контакт.

Собирают таким образом схему, а потом переключатели у них почему-то некорректно работают (зависят друг от друга).

Запомните, что на разных выключателях общий контакт может быть где угодно!

И лучше всего вызванить его, что называется “вживую”, тестером или индикаторной отверткой.

Чаще всего с такой проблемой сталкиваются при монтаже или замене проходных переключателей от разных фирм. Если раньше все работало, а после замены одного схема перестала работать – значить перепутали провода. Но может быть и такой вариант, что новый переключатель вовсе и не проходной. Также запомните, что подсветка внутри изделия никак не может влиять на сам принцип переключения.

Еще одна распространенная ошибка – неправильное подсоединение перекрестных. Когда оба провода, с проходного №1 сажают на верхние контакты, а с №2 на нижние. А между тем у крестового выключателя схема и механизм переключения совсем иной. И подключать провода нужно крест-накрест.

Использование схем с тремя выключателями

Переключающий механизм в проходных выключателях расположен по центру контактов. Устройство обеспечивает комфорт управления светом, безопасность перемещения людей. При такой организации питания, одинаковые устройства не подойдут. Чем больше приборов участвуют в реализации системы управления, тем сложнее получается схема построения.

Полезное видео Выключатель с трех мест — современное решение управление светом Электроэнергия и прочие ресурсы растут в цене, а появление современных технологий позволяет значительно экономить. Многие начинающие электрики не профессиональные путают эти понятия, и пытаются организовать проходную схему на трехклавишнике. Берется отвертка с фазоискателем или мультиметр, и ищется где плюс, а где минус. Речь идет о длинных коридорах, лестницах, подвальных помещениях.

Допустим, один из них будет находиться возле кровати, второй у выхода комнаты, а третий возле рабочего стола. Внешний вид дублирующих устройств почти такой же, как и у одноклавишного прибора. Трехклавишный проходной выключатель Такой коммутатор на самом деле проходным не является, и не может быть использован в схеме освещения с несколькими точками включения. Если вам нужна схема подключения проходного выключателя с двух мест , смотрите ее в данной статье.

При монтаже переключатели устанавливаются так, чтобы в выключенном виде клавиши находились в одном направлении. В нем присутствует и вход, и выход в количестве 2. А между тем у крестового выключателя схема и механизм переключения совсем иной. По правилам прокладывания проводки, все провода в такой электрической цепи должны быть расположены на расстоянии 15 см от потолка. Включение осветительного прибора осуществляется в том случае, когда 2 переключателя встают в одинаковое положение.

Аналогичное крепление оставшихся выходов. А два других установлены на каких-то дворовых постройках гараж, сарай, дойдя до которых можно отключить освещение. В быту используются не только понижающие, но и повышающие трансформаторы. В многоквартирных домах на три этажа.

Пример работы освещения

  1. При включении клавиши №1, лампа горит, электричество идет по фазному проводу и обозначается буквой L, а ход тока показывает красная линия.
  2. Обратное нажатие клавиши, свет гаснет.
  3. Переключаем переходной отсоединитель, лампа загорается.
  4. Нажимаем клавишу снова лампа выключается.
  5. При включении устройства №3 лампа горит. 
  6. Повторное нажатие приводит к выключению светильника.

Рекомендация: при необходимости увеличить количество мест управления светильником добавляется желаемое количество перекрестных выключателей между лестничными.

Необходимое оборудование и материалы

  • Выключатели
  • Монтажная коробка
  • Электроизоляционная лента
  • Клеммы
  • Отвертки крестовые и обычные
  • Нож для монтажа
  • Бокорезы
  • Плоскогубцы
  • Ключи гаечные
  • Электрокабель

Если в помещение уже проведена проводка, и нужно установить дублирующие выключатели, тогда нужно сделать штробы или открытый монтаж кабелей. Для того чтобы сделать штробы понадобиться перфоратор и штроборез. Еще нужен алебастр. Он будет крепить гофтрубу. В случае открытого монтажа необходима распределительная коробка, с помощью гофтрубы она крепится на стену.

Пошаговая инструкция монтажа

  1. Отсоединить электроэнергию в помещении.
  2. Определить, где находятся провода, дабы не повредить их.
  3. Обозначить будущее месторасположение распределительной коробки.
  4. Установить монтажную коробку.
  5. Прокладка электрокабелей. Лучше взять 3-х или 4-х жильный кабель. Для перекидных устройств нужен трехжильный. С помощью одной жилы будет подключаться подача фазы или лампа. Две жилы соединяются с промежуточными проводами. Для перекрестного устройства нужен четырех жильный кабель — по две жилы на каждый выключатель. Две будут вести к первому, а остальные две ко второму.

Концы всех кабелей ведутся в монтажную коробку и соединяются клеммами. А ноль идет к светильнику. Для оборудования проходного выключателя с управлением с 3х мест необходимо иметь навыки и точную схему подключения. Ее наличие дает возможность провести правильную и качественную систему освещения. А на ее основе с легкостью можно создавать более сложные схемы иллюминаций.

Видеоинструкция – Подключение проходного выключателя и управление освещением из 3х мест

Источники

  • https://220.guru/electroprovodka/rozetki-vyklyuchateli/sxema-podklyucheniya-proxodnogo-vyklyuchatelya-s-3x-mest.html
  • https://elektrika.expert/vykljuchateli/vykljuchatel-s-treh-mest.html
  • https://domikelectrica.ru/kak-pravilno-podklyuchit-2-proxodnyx-vyklyuchatelya/
  • https://StrojDvor.ru/elektrosnabzhenie/kak-podklyuchit-proxodnoj-vyklyuchatel-na-tri-tochki/
  • https://tokzamer.ru/bez-rubriki/prohodnoj-vykljuchatel-na-3-tochki-shema-podkljucheniya
  • https://electricvdele.ru/elektroprovodka/rozetki-i-vyklyuchateli/shema-podklyucheniya-prohodnogo-vyklyuchatelya-s-3h-mest.html

Схема подключения проходного выключателя с 3х мест: проходной перекрестный выключатель

Для более удобного управления освещением в частном доме или квартире используются схемы подключения проходных выключателей с 3х мест. В этом случае необходим проходной перекрёстный выключатель. Для монтажа необходимо знать схемы подключения. Надо уметь правильно соединять провода в распределительной коробке. Для этого понадобятся принципиальная и монтажная схемы. При профессиональном монтаже: обязательна сварка всех скруток!
Для включения и выключения освещения с 3х мест и состоящего из светильника или группы светильников используются два одноклавишных проходных выключателя и один одноклавишный проходной перекрёстный. То есть, из трёх проходных, необходим один специальный проходной выключатель, по внутренней конструкции это различные устройства!
Такая схема управления освещением удобна, например, в спальне. Один выключатель расположен на входе, второй с одной стороны двуспальной кровати и третий с другой стороны кровати.
Принципиальная схема состоит из двух проходных (маршевых, лестничных) выключателей, которые по принципу своей работы являются переключателями, одного, так называемого, перекрёстного (промежуточного) выключателя, в конструкции которого специальная схема перекрёстного переключателя, светильника, магистрали и ответвлений фазного, нулевого и нулевого защитного проводников:

Скачать принципиальную схему подключения проходного выключателя с 3х мест.

Монтажная схема соединений в распределительной коробке:

Скачать схему соединений в распределительной коробке для двух проходных выключателей, одного перекрёстного и светильника.
Порядок действий при сборке распределительной коробки с пятью проводами — проводом питания, тремя проводами на выключатели и проводом на светильник. Провод на перекрёстный (промежуточный) выключатель четырёхжильный, все остальные провода трехжильные.

  1. Зачистить, скрутить и заизолировать белую жилу провода питания с белой жилой провода на выключатель №1.
  2. Зачистить, скрутить и заизолировать синюю жилу провода на выключатель №1 с синей жилой провода на перекрёстный выключатель.
  3. Зачистить, скрутить и заизолировать жёлтую жилу провода на выключатель №1 с белой жилой провода на перекрёстный выключатель.
  4. Зачистить, скрутить и заизолировать жёлтую жилу провода на перекрёстный выключатель с синей жилой провода на выключатель №2.
  5. Зачистить, скрутить и заизолировать красную жилу провода на перекрёстный выключатель с жёлтой жилой провода на выключатель №2.
  6. Зачистить, скрутить и заизолировать белую жилу провода на выключатель №2 с белой жилой провода на светильник.
  7. Зачистить, скрутить и заизолировать синюю жилу провода питания с синей жилой провода на светильник.
  8. Зачистить, скрутить и заизолировать жёлтую жилу провода питания с жёлтой жилой провода на светильник.
  9. Уложить в коробку скрутки и закрыть крышкой.

Похожие статьи

  1. Схема подключения проходного выключателя в распределительной коробке.
  2. Схема подключения двухклавишного проходного выключателя в распределительной коробке.
  3. Схема подключения двухклавишного выключателя в распределительной коробке.

Подключение перекрестного выключателя с трех мест. Разновидности выключателей и схемы их подключения

Для удобства пользователей особенно в помещениях с большой площадью: спортивных или концертных залах, длинных коридорах хорошо иметь возможность выключать и включать свет из разных мест. Это избавит от лишних переходов из одного конца помещения в другой.

Двухконтактный проходной выключатель

Схема подключения двухклавишного проходного выключателя легко решает такую задачу, она позволяет установить выключатели в двух и более местах, откуда можно управлять освещением.

Устройство и принцип работы

В основе функциональности и практичности двухклавишного проходного выключателя заложены принципы работы одноклавишного переходного выключателя.

Практически в одном корпусе собрано два одноклавишных проходных выключателя, что значительно расширяет его возможности и всей схемы электропроводки.

Принцип перекидывания контактов остался прежний: на выключателе две входные клеммы, 4 выходные, всего 6 штук.

На клавишах отмечены стрелками, куда нажимать для включения или выключения света. Схема собрана так, что выключить свет можно нажатием любой клавиши на проходных выключателях. В каком положении находятся клавиши используемого проходного выключателя, не существенно. Возможности двухклавишных проходных выключателей позволяют создавать различные варианты схем управления светом из двух и даже из трех мест.

Управление с двух мест

Два двухклавишных проходных выключателя в схеме расположены в разных местах, работают автономно на два направления, управляя разными группами световых приборов.

Схема очень удобна для длинных тоннелей, коридоров, лестничных маршей. Не имеет значения, с какой стороны вы заходите, всегда можно включить свет, а на другом конце при выходе выключить.

Выбираются места установки двухклавишных проходных выключателей, логично их устанавливать на разных концах объекта. Специальной коронкой с победитовыми или алмазными зубцами в бетонных или кирпичных стенах сверлятся установочные отверстия диаметром 72 или 80 мм в зависимости от выбранного корпуса выключателя.

В отверстия вставляются металлические или пластиковые коробки цилиндрической формы (подрозетники). Если проводка наружная, то корпус выключателя крепится к стене дюбелями и саморезами.

По всей длине помещения, на потолке или стенах, устанавливают две группы осветительных приборов. Это могут быть люстры, дешевые светильники или бра. Подключения выполняются по параллельной схеме, чтобы при неисправности одного светильника остальные продолжали работать. От них до распределительной коробки прокладывают провода, для работы схемы достаточно по две жилы в кабеле.

По требованиям ПУЭ (правил устройства электроустановок), в целях безопасности корпуса осветительных приборов заземляются. Все современные светильники, люстры и другие приборы оборудованы клеммой заземления.

Поэтому лучше проложить кабель с тремя жилами:

  • L – фаза красного цвета;
  • N – нулевой рабочий провод синего или черного цвета;
  • заземляющий провод желто-зеленой раскраски.

В конструкции двухклавишного проходного выключателя предусмотрено 6 клемм для подключения проводов, поэтому от каждой коробки под выключатели до распределительной коробки нужно проложить два трехжильных кабеля, оставляя концы по 15-20 см для зачистки изоляции и соединения проводов.

Подключение двухконтактного переключателя

В распределительную коробку должны сойтись 7 кабелей:

  • 4 от выключателей;
  • 2 от групп освещения;
  • 1 кабель питания.

21 – провод, все разводятся на 8 контактов. Требуется сделать расключение согласно указанной схеме.

Схема управления освещением из двух мест

Требование соответствия проводов цвету по назначению L; N, земля, можно выполнить только на участках цепи кабеля питания и осветительных групп. В интервалах от выключателей до распределительной коробки они не выполнимы, в этом случае используются любые провода.

Поэтому надо быть очень внимательным, для выявления нужного провода используйте мультиметр в режиме прозвонки. Один провод в распределительной коробке останется незадействованным – от выключателя с правой стороны.

Заизолируйте его концы и оставьте как резервный или подключите на заземляющий контакт.

Управление в трех местах

В этой схеме используются те же элементы, что для управления переключениями из 2-х точек, плюс перекрестный двухклавишный выключатель. Размещается он в любой удобной точке помещения по схеме между двумя двухклавишными переключателями.

По конструктивному исполнению и принципу действия перекрестный двухклавишный переключатель представляет собой два проходных двухклавишных выключателя в одном корпусе. Его можно заменить двумя проходными двухклавишными выключателями, но практичнее использовать заводской в одном корпусе.

В этой модели на двухклавишный переключатель установлена одна общая клавиша, которая синхронизирует перекидывание контактов двух линий. На клеммах перекрестного двухклавишного переключателя поставлены перемычки таким образом, чтобы он обеспечивал прохождение тока в нужном направлении.


Перекрестный двухклавишный выключатель

Крайние проходные двухклавишные выключатели соединяются с перекрестным четырехжильным кабелем. Схема работает следующим образом: рабочий ноль заводится сразу на две группы освещения. Фаза приходит на оба входных контакта первого двухклавишного проходного переключателя.

Независимо от положения клавиш ток проходит на 2 из четырех входных контакта перекрестного переключателя через перемычку на вход второго проходного двухклавишного переключателя. В данный момент положение клавиш 2-го переключателя определяет прохождение тока на одну из осветительных групп.

Если группа светится, достаточно изменить положение клавиши любого из переключателей этого ряда, чтобы оборвать цепь питания. Точно так же с включением: стоит изменить положение любой клавиши в этом ряду, и цепь будет восстановлена.


Схема управления освещением из трех мест

Конструкции проходных выключателей значительно расширяют возможности переключения осветительных цепей. Они универсальны, их можно использовать как обычные выключатели с одной клавишей или как двухклавишные переключатели.

Как недостаток можно отметить стоимость: они дороже, чем простые модели переключателей. Поэтому использовать их рекомендуется по назначению в схемах с проходным управлением освещением из нескольких мест.

Как подключить. Видео

Ознакомиться с особенностями подключения проходного выключателя можно в представленном ниже видео. Соблюдение выработанной технологии поможет избежать непредвиденных последствий в дальнейшем.

Представленные схемы, фотографии и информация, изложенная в тексте, позволят желающим разобраться в принципах работы двухклавишных проходных выключателей, понять рациональность практического применения схем и их использования, освоить методику самостоятельной установки систем управления освещением из нескольких мест.

Это даст возможность не обращаться к профессиональным электрикам и сэкономить значительные средства при электромонтажных работах в собственном доме.

Приветствую всех читателей моего ! В очередной статье я расскажу по многочисленным просьбам как управлять освещением с двух, трех, четырех, пяти и т.д. мест.

Сейчас я покажу более сложную схему для того, что бы управлять освещением с трех и более мест.

Это можно сделать например с помощью перекрестных переключателей. Что это такое и как они выглядят? Но давайте обо всем по порядку.

Где может в доме понадобиться включать свет из трех мест?

Да в принципе где угодно, например в спальне у каждой прикроватной тумбочки установить выключатель плюс выключатель около двери.

Зашли в спальню, включили свет около двери, затем легли спать и выключили свет у прикроватной тумбочки- согласитесь что это удобно.

Еще вариант- освещение длинного коридора, тогда можно условно разделить его на три участка и в начале каждого участка поставить выключатель.

Или еще способ- освещение подъезда в трехэтажном доме. Зашли в подъезд- включили свет, поднялись на свой этаж- выключили. Жители подъезда могут на любом этаже включать и выключать подъездное освещение.

Важное примечание: освещение в этом случае будет включаться/отключаться одновременно на трех этажах!

Если же потребуется управлять каждой лампочкой по отдельности с любого этажа (например на первом этаже управлять лампой третьего этажа или на втором- первого этажа и т.п.) то придется на каждую лампу собирать отдельную схему управления с трех и более мест.

Да, кстати, схема для управления освещением с трех мест универсальная, ее можно легко продлить для управления с четырех, шести, десяти и более мест))) Но об этом чуть позже, а пока я хочу начать с повторения- с более простой схемы-

Управление освещением с двух мест с помощью проходных выключателей

Внешне проходные выключатели, а правильное их название проходные переключатели выглядят как обычный одноклавишный выключатель.

А почему- переключатель? Тут дело в том, что это устройство при любом положении клавиши не разрывает электрическую цепь, а только переключает с одного контакта на другой. потому и- переключатели .

Вот типовая схема управления освещением с двух мест с помощью проходных переключателей:


При нажатии на клавишу любого переключателя можно включить/выключить лампу независимо от того в каком положении находится другой переключатель.

Фазный провод у меня показан красным цветом, нулевой- синим, переключатели для удобства подписаны №1 и №2.

При нажатии на клавишу переключателя №2 лампочка погаснет, так как в нем при этом “рвется” фазный провод, в том месте где кончается красная линия (зеленая стрелка показывает в какую сторону двигается контакт):


После этого нажимаем на клавишу проходного переключателя №1 и включаем лампу- путь прохождения электрического тока по фазному проводу обозначен красной линией (так будет на всех рисунках ниже):


Нажимаем клавишу проходного выключателя №2, контакт перекидывается вверх и гасит лампу освещения:


Затем нажимаем переключатель №1, его контакт перекидывается вверх и включает лампочку:


Так работает схема проходного выключателя для управления освещением с двух мест. Запомнить ее в принципе не сложно, несмотря на ее кажущуюся сложность.

Надо главное найти на переключателе общую клемму контакта, то есть ту клемму, в которой он не переключается и где контакт зафиксирован одной стороной.

Найдя эти клеммы на обоих переключателях просто к одному переключателю подключаем на эту клемму фазный провод, а ко второму- провод от лампочки.

А две оставшиеся клеммы между переключателями соединяем в любой последовательности- без разницы. Нулевой провод как обычно в схеме выключателя идет на лампочку напрямую через распредкоробку.

Итого в распредкоробке у этой схемы проходного выключателя будет 5 соединений проводов.

Кстати проходные переключатели бывают еще и двойные- то есть в одном корпусе размещены два отдельных независимых проходных переключателя, выглядит как обычный двухклавишный выключатель и имеет шесть клемм.

Управление освещением с трех и более мест

Для этого понадобится как я уже упоминал перекрестный переключатель. Фотографию я его показывать не буду- так как на вид это тоже самый обычный одноклавишный выключатель.

Единственное внешнее отличие- четыре клеммы на обратной стороне для подключения проводов.

Так же как и двойные переключатели- перекрестные переключатели тоже есть двойные, для подключения проводов у них восемь клемм.

Итак, для того чтобы управлять освещением с трех мест понадобится два проходных переключателя и один перекрестный.

Проходные переключатели устанавливаются в начале и конце линии, а перекрестный- между ними, вот схема подключения проходных и перекрестного переключателей:


Почему перекрестный переключатель так назван? Дело в том, что через этот переключатель проходят две независимые электрические линии и он переключает их в крест.

Что бы это понять я сделал два рисунка. Рисунок первый- перекрестный переключатель соединяет электрические линии напрямую, в параллель:


А вот на этой схеме- электрические линии перекрещиваются между собой, отсюда и название- “перекрестный”:


Ну а сейчас подробнее-

Как работает схема управления освещением с трех мест с помощью проходных и перекрестного переключателей

Перекрестный переключатель обозначен буквой икс (Х). Работа схемы обозначена по аналогии с вышеописанной схемой проходных выключателей.

Представьте что это управление освещением в подъезде трехэтажки. Проходной переключатель №1 установлен на 1 этаже, перекрестный переключатель- на 2 этаже, а проходной переключатель №2- на третьем этаже.

Итак, включаем свет (нажимаем клавишу переключателя №1)- лампочка горит, электрический ток по фазному проводу проходит как нарисовано красной линией:



Поднимаемся на второй этаж и проверяем перекрестный переключатель-нажимаем клавишу, включается свет:


Нажимаем клавишу обратно, выключаем свет:


Поднимаемся на третий этаж ко второму проходному выключателю, нажимаем у него клавишу- включается свет:


Оставляем проходной переключатель №2 в этом положении, спускаемся на 2 этаж и нажимаем клавишу перекрестного переключателя- выключаем свет:


Опять же- оставляем в таком положении перекрестный переключатель и спускаемся на первый этаж, нажимаем клавишу первого проходного переключателя- свет включается:


Вот таким образом и работает схема управления освещением с трех мест с помощью проходных и перекрестного переключателей.

При такой схеме в распредкоробке уже будет 7 соединений.

Если необходимо управлять освещением не с трех, а с четырех, пяти и более мест, то для этого просто добавляют еще необходимое количество перекрестных переключателей между проходными, вот и все!

Например вот как на этой схеме что я нарисовал:


Если же управлять каждой лампочкой с любого этажа- то придется устанавливать по три выключателя на каждом этаже- на первом и третьем этаже по три проходных выключателя, а на втором этаже- три перекрестных выключателя.

И собирать три таких схемы- по одной схеме на каждую лампу. Можно сделать и по одному двойному, одному простому проходному выключателю на первом и третьем этаже, а на втором сделать так же один двойной перекрестный и плюс одинарный перекрестный- в этом случае на каждом этаже будет по две установочные коробки под выключатели.

Но схемы собирать все равно придется три)))

На этом у меня все, надеюсь понятно объяснил схемы проходных выключателей?

Напоследок- видео по теме

“Как найти общую клемму (зажим) у проходного выключателя”

Буду рад вашим комментариям, если есть какие то технические вопросы- то прошу задавать их на форуме, именно там я отвечаю на вопросы- .

Подписывайтесь на мой канал на Ютубе ! Смотрите еще много видео по электрике для дома!

Узнайте первым о новых материалах сайта!

Проходные электровыключатели, по сравнению с обычными, предназначены для сложного и комфортного управления освещением. Они позволяют управлять одним светильником из нескольких мест. Например, у вас в квартире длинная комнаты: вы пришли домой после работы, включили свет на входе, разделись и пошли на кухню, а на другом конце прихожей выключили свет вторым выключателем.

Удобство такого управления в отсутствии необходимости в возвращении на исходную позицию для отключения освещения. Еще совсем недавно проходные коммутаторы использовали только в протяженных конструкциях – коридорах, лестницах, на дачных участках. Сейчас их устанавливают везде. Например, в спальне, один электровыключатель устанавливается на входе, а второй – возле кровати. Вошел – включил свет, лег в кровать – выключил.

Принцип работы и терминология

Понятие «проходной выключатель» не совсем обозначает его функциональную принадлежность. Если его применять по прямому назначению, то логичнее употреблять термин «проходной переключатель». Но в обиходе прижились оба варианты.

Проводной выключатель внешне ничем не отличается от стандартного. Разница только в организации системы контактов. Для начала вспомним как работает обычный выключатель. Основное его назначение – это размыкание и замыкание цепи. Назначения переключателя, в принципе, также.

Разницу можно заметить, если взглянуть на условные обозначения.

На рисунке видно, что переключатель оснащен третьим контактом, назначение которого разительно отличается от двухклавишного выключателя, который также имеет три контакта.

Фактически, разница работы выключатели и переключателя происходит в момент размыкания электрической цепи:

  • выключатель разрывает цепь;
  • переключатель разрывает одну цепь и замыкает вторую.

Вторая цепь в данном случае – это контакты парного переключателя, так как эти устройства не работают самостоятельно.

Рассмотрим схему подключения проходных переключателей с управлением из двух мест.

Принцип работы контактов такого выключателя похож на коромысло. На рынке, кстати, редко, но все же встречаются переключатели с нулевым положением, то есть разомкнутыми могут быть сразу две цепи. Проходной коммутатор можно использовать как обычный – третий контакт в таком случае не подключается.

Второй вариант, когда из обычного выключателя делается проходной, рассматривать не будем. Это было характерно для 90 лет, но сейчас на рынке представлено достаточное количество моделей по приемлемым ценам и экспериментировать в данном направлении не совсем целесообразно.

Виды переключателей

На рисунках ниже представлены функциональные схемы различных проходных выключателей.


Самые распространенные варианты:

  • одноклавишные;
  • двухклавишные;
  • трехклавишные.

Также популярностью пользуется перекрестный электровыключатель, используемый для управления из трех и более мест и представляющий собой два одноклавишных переключателя, спаренных внутренними перемычками. Перекрестный переключатель можно изготовить из двухклавишного электровыключателя, установив внешние перемычки. Перекрестник имеет одну клавишу, перекидывающую одновременно две контактных группы, а у двухклавишного – каждая клавиша управляет своей группой.

Схемы включения в электрическую сеть

Существует несколько схем подключения переключателей. Использование конкретной схемы зависит от количества управляемых светильников и количества мест управления.

Схема управления с двух мест

Для начала рассмотрим самую популярную схему – управление с двух мест.

Для работы переключателей понадобиться прокладка трехжильной проводки к каждому устройству. Распределительная коробка монтируется напротив одного из выключателей. В нее заводятся следующие провода:

  • кабель питания;
  • кабель одного выключателя;
  • кабель второго выключателя;
  • кабель осветительного прибора.

Также может быть еще один кабеля отходящей линии, но в простой схеме он не учитывается.

Посмотрим наглядно схему подключения в распределительной коробке.

Схема управления двумя светильниками

Данная схема позволит управлять двумя светильниками с двух мест. Она практически аналогична предыдущей, за исключением использования двухклавишных выключателей.

Для экономии можно установить внешнюю перемычку для второй секции выключателя, а не прокладывать два провода к первому выключателю.

Схема управления из трех мест

Для реализации такой схемы понадобится пара одноклавишных выключателей и перекрестный переключатель. Для работы перекрестника потребуется четырехжильный кабель, либо два двухжильных.

Схемы управления из четырех и более мест

Если в предыдущую схему добавить еще один перекрестный переключатель, то управлять освещением можно из четырех мест. Включается он последовательно с первым перекрестником.


Схема управления двумя светильниками из трех мест


Такая схема имеет множество недостатков – большое количество соединений и множество хлопот по маркировке жил кабеля.

Во всех рассмотренных схемам показано начальное положение контактов переходных выключателей.

На этом собственно и все, что следует знать начинающему электрику.

Выключатель и переключатель – сегодня важная функциональная деталь в каждом доме, без которой трудно представить себе любое помещение. Иногда начинающие электромонтажники путаются в терминологии, в схемах и принципах работы этих двух, а точнее трех механизмов (т.к. переключатели тоже бывают двух видов). Не говоря уже о простых покупателях, которые сами пытаются смонтировать, либо купить для дальнейшего монтажа нужные устройства. В этой статье мы постараемся пролить свет на разницу между выключателем и переключателем.

Итак, выключатели и переключатели служат для коммутации электрических цепей освещения и бытовых приборов, внешне также они выглядят одинаково, разница лишь в количестве контактов с тыльной стороны. Но выключатель предназначен для разрыва одной цепи, а переключатель для переключения между цепями. Выключатель применяется для управления светом из одного места, переключатели же служат для управления светом из двух и более мест, при чем для реализации управления с трех мест и более используются «проходные» переключатели.

Выключатель – устройство, обычно имеющее два контакта, которое во включенном состоянии соединяет контакты (включает лампу), а в выключенном состоянии соответственно, разъединяет контакты (выключает лампу). Здесь все очень очевидно и понятно.

Переключатель — устройство, имеющее три контакта (или более). Во «Включенном состоянии» замыкает первый и второй контакты, а в «Выключенном состоянии» замыкает первый и третий контакты. По сути, переключатель постоянно находится во включенном состоянии – либо в одном, либо в другом. Отсюда и название «Переключатель» — переключает с одного контакта на другой.
Если у переключателя задействовать только два контакта, он будет работать как выключатель.
Одно из самых популярных применений переключателя – это управление освещением с двух мест. Для управления освещением потребуется всего два переключателя, а для управления освещением с трех и более мест не обойтись без применения промежуточных (перекрестных) переключателей.

Промежуточный (он же перекрестный) переключатель – устройство, переключающее две отдельные линии накрест (то есть, если до перекрестного переключателя фаза шла справа, а ноль слева, то при переключении они поменяются местами). Внешний вид промежуточных переключателей ничем не отличается от обычных выключателей.

Обычно применяют для управления освещением с трех и более мест. Этот переключатель называют «перекрестным», потому что он при переключении как бы перекрещивает линии. А «Промежуточным» называют за то что он в схеме включения при управлении с трех или более мест находится в промежутке между «переключателями на два направления».

Технические характеристики переключателей Legrand Valena
  • Распорные подпружиненные лапки механизмов оснащены универсальными винтами и надежно спрятаны, что упрощает установку и предотвращает повреждение рук об острые края при монтаже.
  • Проходной выключатель имеет глубину механизма всего 21 мм, что облегчает установку в монтажную коробку и подвод к нему проводников.
  • Механизм проходного выключателя надежно закреплен на металлическом суппорте в четырех точках.
  • На тыльной стороне механизма проходного выключателя расположен специальный маркер для замера длинны снятия изоляции с кабеля.
  • Четкая маркировка и схема подключения, нанесенные на механизмах розеток и выключателей, делают процесс правильного подключения простым и понятным.
  • имеет мощные клеммы, способные надежно держать как алюминиевый, так и медный кабель.
  • Быстрое и безопасное подключение кабеля с помощью автоматических клемм — автоматические клеммы для подключения выключателей и переключателей, расположены под углом 35º для упрощения фиксации кабеля и обеспечения надежности контакта.
  • Легкая центровка нескольких механизмов в ряд — оцинкованный металлический суппорт с технологическими пазами и отверстиями, позволяет точно установить все механизмы в многопостовой рамке.
  • Возможность замены индикатора подсветки без демонтажа механизмов.
  • Ресурс выключателей и переключателей соответствует требованиям европейских стандартов и составляет 40 000 коммутаций.
  • Проходной выключатель гарантированно коммутирует 10 А при 230/250 В~.

Выключатели и розетки — важная деталь электрооборудования помещения. Ведь, чем лучше контакт при включении выключателя, между розеткой и вилкой, тем меньше он будет нагреваться и соответственно снижается вероятность возгорания.
Ниже идёт список популярной электрофурнитуры
Легранд (Legrand)
АББ (AВВ)
Поло (Polo)
Вико (Vi-ko)
Макел (Makel)

Разновидности выключателей и схемы их подключения

Существует несколько видов выключателей и соответственно, различных типов подключений:
,
,
,

Также существуют выключатели с подсветкой, правда, при подключении такого типа выключателей желательно обратить внимание на тип источника освещения, например, люминесцентные лампы в цепи с выключателем с подсветкой, в выключенном состоянии будет слегка светиться.

Одноклавишный выключатель. Схема подключения

Самая простая цепь у одноклавишного выключателя и регулятора мощности.



Одноклавишный выключатель имеет всего два контакта и что-нибудь в нём перепутать достаточно сложно. Коммутируется цепь так: Находим провод питания: ноль и фазу, затем нулевой провод подаём сразу на лампу (светильник, люстру), а фазный — на выключатель. После этого, от выключателя прокидываем провод к источнику света. Обычно цепь коммутируется через распредкоробку, где провода скручиваются должным образом.


Регулятор мощности также имеет два контакта, с той лишь разницей, что этот регулятор не просто включает и выключает свет, а плавно его регулирует.
Прошу обратить внимание, что не все лампы работают с этим устройством. Поэтому, перед монтажом регулятора мощности желательно уточнить это у электрика или продавца.

Двухклавишный выключатель. Схема подключения

Двухклавишный выключатель работает как два одноклавишных.
Единственная разница между цепью двух одноклавишных и одного двухклавишного есть отсутствие дублирующих проводов.


Т.е. не имеет смысла протягивать две «нейтрали» к лампам, которые расположены рядом.
Двухклавишный выключатель имеет три контакта. Образно говоря, один вход и два выхода. Здесь важно найти общий (входящий) контакт, а то подключив неверно, лампы будут включаться неправильно. Важно знать где вход, а перемена проводов на выходе лишь изменит клавиши, которые отвечают за включение той или иной группы ламп.

Проходной выключатель. Схема подключения

Правильнее было бы его назвать не выключатель, а переключатель. Суть его в том, что он переключает подачу тока с одного провода на другой.Как правило, их используют в паре. Проходные выключатели, часто используют в длинных коридорах, лестничных пролётах и пр. Смотрите схему, думаю, на ней всё понятно.



Проходной выключатель имеет также три контакта, как и двухклавишный, только, как видите, работает по-другому. В нём, как и в двухклавишном, есть один входной контакт и два выхода. Важно знать где вход, а перемена проводов на выходе лишь изменит положение клавиши. При неправильном подключении проходных выключателей, цепь будет работать неправильно.

Перекрёстный выключатель. Схема подключения

Этот тип выключателей используется в случае, если требуется включение-выключение одного источника света с трёх и более мест, реже- для переключения подачи полярности электропитания.В первом случае его используют вместе с двумя проходными. То есть, цепь из выключателей должна начинаться и заканчиваться проходным выключателем, а между ними можно установить теоретически бесконечное количество перекрёстных.



Перекрёстный выключатель имеет четыре контакта: два входных, два выходных. Суть переключения в том, что в одном положении первый входной провод замкнут с первым проводом на выходе, второй входной замкнут со вторым выходным проводом, в другом положении, провода как бы, меняются местами т.е. первый входной замкнут со вторым выходным, второй входной- с первым выходным. Здесь важно не перепутать и обращать внимание на обозначения, а то цепь работать не будет правильно.

Установка выключателя

Схема подключения проходного выключателя с 2-х, 3-х и 4-х мест

Стандартная ситуация: вы вошли в дом и включили свет в коридоре, а затем переместились в спальню. Теперь нужно вернуться, чтобы потушить коридорную лампочку, что не слишком удобно. А если комната отдыха расположена на втором этаже частного дома, то вам придется дважды преодолеть лестницу, чтобы вырубить освещение в прихожей. Решить эту проблему поможет схема подключения так называемого проходного выключателя, позволяющая задействовать управление одним светильником (или группой ламп) с 2-х мест.

Несмотря на кажущуюся простоту конструкции, настоятельно рекомендуем обращаться помощью к специалисту, а именно электрику, электрика в Запорожье можно найти на сайте elektromaster.zp.ua. Работник обязательно проконсультирует по всем вопросам, касающихся проводки и проведет качественную работу, безопасно для Вашей жизни.

Принцип управления из двух точек

На практике система работает следующим образом:

  1. Войдя в темный коридор, вы зажигаете осветительный прибор.
  2. Переместившись в другую комнату либо на второй этаж, вы гасите его вторым выключателем, установленным в этом помещении.
  3. Каждый, кто зайдет в дом после вас, сможет опять зажечь свет в прихожей и потушить его удобным способом в одной из двух точек.

Примечание. С таким же успехом можно организовать управление освещением с 3-х или даже 4-х разных мест, о чем будет сказано далее.

Проще говоря, светильник включается и отключается в первой точке независимо от положения клавиши во второй и наоборот. Ключевым элементом схемы является проходной (иначе – маршевый) выключатель, который отличается от обычного одноклавишного тремя контактами для подсоединения проводки. Два таких устройства нужно поставить в удобных местах и подключить к электрической сети тройным (трехжильным) кабелем по следующей схеме:

По сути, наши устройства представляют собой переключатели, перекидывающие фазный ток по одному из двух направлений. Между этими линиями и происходит переключение, только с разных сторон. В каком бы положении ни находились кнопки изначально, нажатие на любую из двух клавиш приведет к замыканию либо разрыву электрической цепи.

На фото видно, что средний контакт замыкается на один из крайних, режим полного выключения отсутствует

Справка. Проходные переключатели – далеко не новинка. Обычные, двухклавишные и трехклавишные модели изделий достаточно давно выпускаются известными производителями электрооборудования — Schneider Electric (Шнайдер Электрик), Legrand (Легранд) и Lezard (Лезард). Как выглядит подобное устройство, показано на фото.

Чтобы управлять из разных комнат группой светильников с возможностью включения одной либо нескольких ламп, нужно использовать двойные (двухполюсные) переключатели и соединить их по такой схеме:

Несколько рекомендаций о том, как правильно подключить провода:

  1. Установите проходные выключатели на пластиковые подрозетники в требуемых местах. От каждого из них проложите в стробах трехжильные кабели к распределительной коробке.
  2. Внутри коробки соедините напрямую нулевой и заземляющий контакт, ведущий к лампочке. Фазные провода от сети и светильника подключите к проводникам, ведущим к перекидным контактам выключателей.
  3. Там же состыкуйте контакты двойной линии между нашими кнопками. На этом монтаж окончен.

Включение из 3-х и более мест

Чтобы реализовать подобное управление освещением, приведенная схема проходного выключателя дополняется еще одним элементом. Это так называемый перекрестный (иначе – спаренный) переключатель на 4 контакта, чья установка предусматривается между крайними отключающими устройствами, как изображено ниже на картинке. Его принцип действия следующий:

  • в первом положении кнопка напрямую замыкает обе цепи;
  • после переключения линии замыкаются крест-накрест.

Примечание. Если необходимо сделать управление светильником из 4-х и более мест, то в схему добавляется второй спаренный переключатель, третий и так далее до бесконечности.

Расключить приборы в данном случае несколько сложнее, так как здесь возникает четырехжильный кабель для подсоединения перекрестного устройства. Распайку лучше делать внутри распределительной коробки, а не в подрозетниках, при этом цвета проводов желательно дублировать бирками, дабы избежать путаницы. Доступно и подробно о схеме подключения рассказывается на видео:

Распространенные ошибки при монтаже

При самостоятельной сборке описанных схем хозяева квартир и частных домов допускают несколько типичных ошибок, отчего система не работает изначально или отказывает в ближайшее время. Перечислим эти недочеты и причины, их вызывающие:

  1. Один из выносных выключателей разрывает цепь окончательно (как правило, перекрестный), остальные тоже перестают функционировать. Это явный признак неверного присоединения контактов, нужно все проверить и подключить правильно.
  2. Одно из клавишных устройств быстро перегорает и его приходится часто менять. Здесь налицо высокая нагрузка от ламп на контакты переключателя, рассчитанные на максимальную мощность 2,2 кВт (ток 10 Ампер). Если ее снизить нельзя, нужно перейти на другой способ коммутации – с помощью импульсных реле с параллельным подсоединением кнопочных выключателей.
  3. Наблюдается периодическое мигание люминесцентных и светодиодных ламп, работающих от проходных переключателей. Причина – низкокачественные изделия с плохой изоляцией (есть утечка) либо дешевые микролампочки ночной подсветки, встраиваемые в корпуса для ориентации в темноте.

Важный момент. Серьезная ошибка, в определенных условиях ведущая к поражению электротоком, — подсоединение к отключающей арматуре нулевого провода вместо фазного.

Кнопочный диммер, совмещенный с проходным переключателем

Также неполадки случаются при использовании одновременно с проходными двухпозиционными переключателями диммеров – электронных устройств для регулировки яркости свечения лампочек. Так бывает, когда вы пытаетесь собрать схему из дешевых элементов с некачественной изоляцией.

Заключение

Невзирая на появление новых способов коммутации освещения с помощью импульсных реле и блоков дистанционного управления, схема с проходными выключателями остается самой простой и доступной по цене комплектующих. Недостаток системы один: у клавиш нет фиксированного положения «вкл» и «выкл», что иногда вызывает неудобства. К примеру, находясь на втором этаже дома, вы не видите, выключен ли свет на первом, а по переключающей кнопке этого не поймешь.

дуг в автоматических выключателях

Контактная дуга в автоматическом выключателе представляет собой чрезвычайно сложный электротермогидродинамический процесс, и мы никогда не можем полностью математически описать детальную физику дуги. Наша цель здесь — разработать приближенную модель дуги, чтобы мы могли рассматривать дугу как элемент цепи и анализировать электрические цепи, содержащие дуги.

Во время нормальной работы выключателя дуга, если она присутствует, постоянно изменяется. Он динамически удлиняется за счет разъединяющих контактов и электромагнитных сил, которые отталкивают его от первоначальной траектории.Он динамически нагревается своим током. Он динамически охлаждается окружающей средой и, возможно, другими вспомогательными средствами (принудительный поток газа, холодные стенки защитной оболочки и т. Д.). И, в зависимости от чистой скорости поглощения энергии (нагрев минус охлаждение), она динамически увеличивается в площади поперечного сечения.

Поскольку дуга изменяется физически и термически, она также изменяется электрически. Изменение электрических характеристик дуги, в свою очередь, изменяет величину сквозного тока, который может подавать внешняя электрическая цепь.Следовательно, инженерное описание (а это все, что мы ищем) дуги выключателя должно включать динамическое описание взаимодействия выключателя и электрической сети. Теперь мы обсудим компоненты электрической дуги: катоды, аноды и плазменные колонны.

Два дуговых электрода называются катодом и анодом. Электроны инжектируются в дугу катодом со скоростью, пропорциональной току дуги. Электроны дуги собираются анодом с той же скоростью, поскольку ток должен быть непрерывным.Область между катодом и анодом разделена на три подобласти: область катодного падения, плазменный столб (иногда называемый положительным столбом) и область анодного падения.

Типичный профиль напряжения вдоль пути «короткой» дуги показан на Рис. 5.7 .

Вкратце, мы подразумеваем, что падение напряжения на плазменном столбе мало по сравнению с объединенными падениями напряжения на катодной и анодной областях падения. Обычно это происходит, когда физическая длина плазменного столба мала.Области катодного и анодного падения являются переходными областями между металлическими катодными и анодными электродами и столбом газовой плазмы. Величины электрических полей внутри катодной и анодной областей падения намного выше, чем величины полей внутри металлического катода и анода. Причем намного больше, чем величина поля в области плазмы. Более высокие электрические поля, по определению, представляют собой более высокие падения напряжения на единицу расстояния, поэтому в описаниях катодных и анодных переходных областей используется термин «падение», как в «падении напряжения».

Падения или «падения» напряжения в областях катодного и анодного спада сильно зависят от материалов, используемых в качестве катодных и анодных электродов, но относительно слабо зависят от уровня тока внутри дуги. Энергия, необходимая для полного удаления электрона с поверхности материального тела, определяется как «работа выхода» этого материала. Выраженная как эквивалентное напряжение (энергия, деленная на заряд одного электрона), работа выхода в вакууме большинства металлических элементов составляет примерно от 4 до 5 вольт.Детальная физика эмиссии и сбора электронов в катодной и анодной областях в условиях дуги настолько сложна, что исследователи теоретически проанализировали лишь ограниченное количество слаботочных упрощенных случаев. Для наших целей достаточно сказать, что катодные и анодные падения напряжения «порядка» работы выхода катода и анода.

Фактическая площадь поверхности катодной электронной эмиссии и анодного сбора электронов зависит от общего тока дуги.Однако плотности тока в этих активных областях чрезвычайно велики, особенно для катода. Плотность тока, превышающая 10 6 А / см 2 , и температура поверхности, превышающая 4000 o К, были постулированы Ли для катодных «пятен». При этих плотностях тока и температуре электронная эмиссия представляет собой комбинацию термоэлектронной и автоэлектронной эмиссии. Электроны с достаточной тепловой энергией могут термоэлектронным способом покинуть поверхность катода, но из-за большой концентрации положительных ионов перед катодом также присутствует сильное поверхностное электрическое поле.Это позволяет поверхностным электронам туннелировать через барьер с уменьшенной поверхностной работой выхода и ускоряться или «испускаться» автоэлектронной эмиссией.

На аноде могут присутствовать даже более высокие поверхностные «точечные» температуры. Когда электроны покидают катод, они забирают с собой энергию. Следовательно, катод фактически охлаждается за счет их выхода (однако в целом катод нагревается за счет нагрева I 2 R внутри катодного пятна и энергии поступающих положительных ионов).Когда электроны попадают на анод, они отводят свою энергию на поверхность анода и нагревают ее (в дополнение к нагреву анода I 2 R).

В зависимости от фактических точечных температур поверхности, испаренный материал поверхности анода и катода будет переходить с более горячих поверхностей на более холодные, если зазор достаточно мал (как в размыкателе при начальном разъединении контактов). Некоторые исследователи показали, что перенос материала может зависеть от пикового тока дуги, когда пиковая температура поверхности передается от катода к аноду, когда пиковый ток дуги превышает определенный порог.

Плазменный столб в дуге состоит из частично ионизированного газа. Молекулы газа «ионизируются», когда молекулы нейтрального газа разделяются на отрицательно заряженные свободные электроны и положительно заряженные ионы. Это происходит в результате ряда различных процессов: столкновений электронов и положительных ионов в сильном электрическом поле; поглощение излучения; и термическая ионизация, ионизация посредством столкновений с высокотемпературными (т. е. высокоэнергетическими) электронами, положительными ионами и нейтральными молекулами. Все эти процессы происходят по дуге; относительная важность каждого из них зависит от расположения в плазменном столбе и силы дуги.Энергия, подводимая к плазменному столбу, представляет собой джоулева нагрев за счет подвижных носителей тока.

Поскольку существует большая разница между массой электрона и массой положительного иона, существует большая разница между откликом электрона и положительного иона на приложенное электрическое поле. Безусловно, большая часть тока в плазменном столбе дуги переносится электронами. Следовательно, первоначальная энергия передается плазме электронному газу внутри плазмы.Но очень быстро за счет столкновений эта энергия распределяется с положительными ионами плазмы и фоновыми нейтральными молекулами. Таким образом, в интервалы времени, представляющие интерес для инженера-проектировщика выключателя, и с очень хорошей степенью приближения плазма находится в состоянии теплового равновесия. То есть все компоненты (электроны, ионы и нейтральные молекулы) в пространственной области имеют одинаковую температуру.

В условиях теплового равновесия скорость ионизации в определенной дифференциальной области уравновешивается равной скоростью ион-электронной рекомбинации.Кроме того, чистые концентрации или плотности электронов и положительных ионов приблизительно равны и монотонно зависят от температуры плазмы.

Проводимость области плазмы в дуге сильно зависит от температуры плазмы. Чем выше температура, тем выше уровень термической ионизации и концентрация носителей. Чем больше носителей, тем меньше значение электрического поля, необходимого для поддержания заданного уровня плотности тока (т. Е. Увеличивается проводимость).Этот эффект положительной обратной связи — больший ток → более высокий нагрев → больше носителей → больший ток для данного уровня внешнего возбуждения — частично объясняет установившееся отрицательное дифференциальное сопротивление дуги.

Еще одним фактором, способствующим установившемуся отрицательному дифференциальному сопротивлению дуги, является распространение плазменного столба в поперечном сечении при более высоких уровнях тока. По мере увеличения температуры активного (ионизированного) плазменного столба увеличивается и температура газа, окружающего плазменный столб, из-за теплопроводности (и, возможно, конвекции и излучения).При достаточно высоких температурах, превышающих пороговую температуру, непосредственно окружающий газ также подвергнется термической ионизации. Тогда будут присутствовать дополнительные носители для переноса тока дуги, дополнительно увеличивая чистую проводимость дуги.

Типичная статическая или установившаяся вольт-амперная характеристика дуги приведена на рис. 5.8 .

Как правило, для данного уровня тока дуги напряжение дуги пропорционально длине дуги. Но для заданной длины дуги более высокие токи дуги приводят к меньшим падениям напряжения дуги из-за статической характеристики отрицательного дифференциального сопротивления.

Общая схема управления дугой, используемая во многих конструкциях автоматических выключателей (т. Е. Метод, используемый для увеличения общего напряжения дуги на главных контактах), заключается в нагнетании дуги в дуговую перегородку или структуру делителя. Типичная конструкция дуговой перегородки в небольшом автоматическом выключателе показана на рис. 5.9. .

Дуговые перегородки разбивают одну дугу на несколько более коротких дуг, соединенных последовательно. Падения напряжения на аноде и катоде этих нескольких дуг складываются и составляют основную часть всего устройства — напряжения дуги.Движение дуги в перегородку инициируется магнитной силой Лоренца, или силой J x B, из-за самого тока дуги (J — плотность тока дуги, а B — плотность магнитного потока, обусловленная током). Эта магнитная сила Лоренца является той же силой, которая стремится отталкивать контактные поверхности друг от друга из-за сужения путей прохождения тока к мельчайшим точкам контакта. Движение дуги из-за этой собственной магнитной силы называется магнитным гашением дуги.

Магнитный обдув также используется для принудительного движения дуги на «бегунках» дуги, которые прикреплены к контактным структурам (см. Рисунок 5.10) .

Использование направляющих дуги позволяет сохранить более дорогой контактный материал из сплава серебра за счет перемещения катодно-анодной «ножки» дуги с контактов на менее дорогой материал направляющих. Кроме того, направляющие дуги представляют собой более длинный путь дуги для прохождения дуги и могут действовать как передающая среда между областью межконтактного контакта и любой областью дуговой перегородки или областью дуговой камеры. Дуговые направляющие можно улучшить, добавив ступенчатые катушки электромагнитного привода, чтобы еще больше усилить силу магнитного выброса по всей длине направляющей. (см. Рисунок 5.11).

При проектировании и анализе выключателя статические характеристики дуги, безусловно, представляют интерес, но главное внимание имеют динамические характеристики дуги. Дуга переносит ток в цепи до тех пор, пока прерывание не будет успешным, то есть будет ли дуга повторно зажигаться при повышении напряжения на контактах выключателя — это вопрос, на который можно ответить только путем изучения динамического поведения дуги.

электрическое — Как получается, что два выключателя управляют одним и тем же?

Глядя на опубликованные вами фото, эти два выключателя связаны с тем, что выглядит как 12/3. Это позволяет двум отдельным цепям совместно использовать одну нейтраль, хотя технически для этого каждая цепь должна быть подключена к противоположным фазам, поскольку две цепи 20 А на одной фазе при максимальной нагрузке будут давать 40 А на нейтрали. Один 20А на каждой фазе поместит только 20А на нейтраль, что и требует код.Однако это не следует исправлять, пока вы не обнаружите, где две цепи взаимосвязаны. Вы подошли ко всем затронутым розеткам и открыли их, чтобы посмотреть на провода в них? Было ли в них и красное, и черное?

Подумайте, как устроен ваш дом. Где находится ваша панель? Что в первую очередь гаснет при выключении автоматов на панели? Судя по возрасту вашей панели и проводки, я бы сказал, что ваш дом, вероятно, датируется примерно 1950-ми годами.Тогда было обычным делом использовать потолочный световой короб в качестве соединения и отделять его от него для питания следующего устройства в очереди. Я жил в доме, который был построен в середине 50-х, оригинальные схемы питали вещи случайным образом по всему дому. Например, одна электрическая цепь питала розетку в гараже, одна розетка в семейном доме, освещение кухни, холла и ванной, а также розетка, управляемая переключателем в каждой из трех спален. Как ни странно, розетки в самой маленькой спальне были отключены от трех разных цепей.При замене светильника в шкафу в главной спальне я обнаружил, что это точка соединения для подачи питания, идущего от панели, а оттуда — к выходным отверстиям и прочему. Я бы посоветовал открыть потолок и распределительные коробки, а также посмотреть в распределительные коробки на встроенных светильниках. Если вы обнаружите этот провод 12/3, проверьте, входят ли несколько проводов в один и тот же зажим / заглушку с ним, и убедитесь, что красный и черный провода в одном корпусе случайно не зацеплены вместе в этой точке.Если вы обнаружите это, разделите провода и включите каждую цепь по очереди, а затем используйте тестер, чтобы проверить, в чем проблема. В этот момент может быть место, где все должно быть разделено, и вы затем можете проверить оставшиеся провода по одному, подключив их к одному из точек доступа на панели, и посмотреть, что происходит, когда вы включаете прерыватель, а затем вы может решить, что должно идти в каждой из цепей. Если у вас есть дверной звонок, найдите трансформатор, установленный на распределительной коробке, и проверьте его, чтобы увидеть, является ли это точкой соединения для провода 12/3, идущего от панели.Посмотрите, что, по вашему мнению, будет самым странным местом для распределительной коробки. В том доме, в котором я жил, когда кухня была переделана, были установлены новые цепи для выходов счетчиков, а одна из оригинальных выходных коробок была повернута лицевой стороной к шкафу с печью, с закрытой плитой, но все еще использовался как распределительная коробка для питания других вещей в доме.

Конечно, не забудьте выключить питание, прежде чем открывать какие-либо коробки. Вам следует снова снять крышку с панели и с помощью тестера убедиться, что питание каждого из этих проводов, выходящих из прерывателей, отключено.Известно, что выключатели в панелях такого типа иногда выходят из строя и показывают, что выключатель выключен, хотя на самом деле внутренние компоненты остаются включенными, а питание все еще включено. Нечасто, но лучше на всякий случай перепроверить.

Сначала вы можете попробовать отключить оба прерывателя и отсоединить черный провод. Включите прерыватель с подключенным красным проводом. Все еще работает на этом выключателе? Затем сделайте обратное: отсоедините красный провод и включите прерыватель черного провода.Как и раньше? Если вы обнаружите, что половина подключенных устройств находится на одном проводе, а остальные — на другом, это может указывать на неисправность внутри прерывателя. Эти два небольших выключателя представляют собой то, что называется сдвоенным выключателем, они содержатся в одном блоке и часто используют одни и те же механические компоненты, и выход из строя одного из них может привести к неправильной работе обоих, в результате чего вы останетесь без защиты в случае перегрузки или короткого замыкания. В этом случае этот сдвоенный выключатель необходимо заменить.

Вам действительно стоит попытаться выяснить и исправить эту проблему.Если вы не можете разобраться в сути, попросите электрика взглянуть на это. По крайней мере, тогда вы можете расслабиться, зная, что опасное состояние было исправлено.

Автоматические выключатели Siemens серии QP

Siemens QP, QT, QAF и QPF — стандартные «сменные» выключатели съемного типа, используемые в центрах нагрузки под несколькими торговыми марками, включая Siemens, Murray, Gould и Sylvania, чтобы назвать наиболее распространенные. Он также подходит для большинства измерительных приборов Midwest Power и Milbank Mfg.Вставные выключатели Siemens очень похожи на GE THQL и в большинстве случаев физически подходят, однако список UL может быть утерян при использовании выключателей Siemens в оборудовании GE. Перед установкой всегда обращайтесь к производителю вашего оборудования, чтобы проверить, какие устройства одобрены.

Автоматические выключатели QP
QT дуплексная, триплексная и четырехплексная версии выключателя QP
QAF версия с отключением от дугового замыкания с выключателем QP
QPF версия с отключением при замыкании на землю выключателя QP
QFGA2 с двумя функциями GFCI и AFCI версия выключателя QP


Автоматические выключатели Siemens QP обеспечивают удобные вставные соединения в корпусах Siemens и экономят время благодаря функции Insta-Wire.Компания Siemens предлагает три класса AIC для этих выключателей: QP @ 10K AIC, QPH @ 22K AIC и HQP @ 65K AIC.

На выбор доступны 4 конструкции:

  • 1 полюс 120 В
  • 2 полюса 120 / 240В
  • 2 полюса 240 В
  • 3 полюса 240 В

Модификации

Калибровка 400 Гц — добавьте суффикс Y

Морская калибровка при температуре окружающей среды 50º C — добавьте суффикс M

Fungus Proofing — добавить суффикс F

Особые приложения

■ HID Lighting — добавить суффикс HID

15 — 30A 1- и 2-полюсные выключатели для нагрузок с газоразрядными лампами высокой интенсивности, имеющие пусковые токи выше уставки мгновенного отключения стандартного выключателя.

■ Лампа накаливания — добавьте суффикс T

15-20A 1-полюсный выключатель, используемый для мощных нагрузок с вольфрамовыми лампами, имеющих пусковые токи выше уставки мгновенного отключения стандартного выключателя.

■ Выключатель в литом корпусе — добавьте суффикс S

1-полюсные выключатели на 100 А и 2-полюсные выключатели на 30, 60 и 125 А для приложений, в которых не требуется защита от сверхтоков.

■ Без шума — добавьте суффикс NN

2-полюсные выключатели на 50–90 А для приложений, требующих снижения 60-тактного гула стандартного выключателя.

■ Переключение нейтрали — добавить префикс QG

15 — 30A 2-х и 3-х полюсные выключатели используются там, где необходимо отключить все проводники. Нейтральный полюс выключателя не подключается к шине центра нагрузки. Одна сторона подключена к нейтрали, а другая — к устройству.

■ Водонагреватель — добавить префикс QW

15 — 30A 3-полюсный выключатель, используемый при установке в центре нагрузки, автоматический выключатель водонагревателя не соединяется с шиной центра нагрузки.К линейным зажимам выключателя подключается отдельная измерительная цепь. Используется в зонах нагрева воды по специальному тарифу.


Выключатели типа QT — это дуплексная, триплексная и четырехплексная версия QP. Они обеспечивают экономию места внутри шкафа за счет установки двух автоматических выключателей 1/2 дюйма на 1 дюйм пространства. Дуплексные выключатели доступны в стандартной версии для центров нагрузки CTL и версии без зажима для центров нагрузки без CTL. Современные центры нагрузки имеют рейтинг CTL и соответствуют разделу 384-15 Национального электротехнического кодекса.Использование выключателей, не соответствующих требованиям CTL, при определенных обстоятельствах может привести к нарушению кода и списков UL некоторых центров нагрузки.

На выбор доступны 3 конструкции:

■ 1 полюс, 120 В, дуплекс

Эти компактные дуплексные выключатели объединяют два независимых полюса выключателя 1/2 «в общий блок. Этот блок подключается к одной стойке центра нагрузки и требует одно место на панели. Рейтинг HACR.

■ 2 полюса 120/240 В, триплекс

Эти трехполюсные выключатели, позволяющие сэкономить место, имеют 2-полюсный общий размыкающий выключатель для цепей 120/240 В переменного тока и два однополюсных для цепей 120 В переменного тока.Триплекс требует двух панельных пространств. Рейтинг HACR.

■ 2 полюса 120/240 В Quadplex

Эти компактные четырехполюсные выключатели обеспечивают два набора двухполюсных выключателей с общим расцеплением для цепей 120/240 В переменного тока и требуют двух мест для панелей. Рейтинг HACR.

СКАЧАТЬ PDF — Автоматические выключатели Siemens типа QT


Тип QAF Прерыватель цепи дугового замыкания (AFCI) с ответвлением фидера — это устройство, предназначенное для устранения сильноточных дуговых замыканий во всей цепи.Сильноточные дуговые замыкания возникают между фазой и нейтралью или фазой на землю. Эти дуговые замыкания происходят параллельно нагрузке.

Тип ответвительного фидера AFCI требуется в соответствии с Национальными электротехническими правилами (NEC) 1999 и 2002 годов. 2005 NEC Раздел 210.12

Жилых спален. Все однофазные розетки на 120 В, 15 и 20 А, питающие розетки, установленные в спальнях жилых домов, должны быть защищены перечисленным в списке автоматическим выключателем дугового замыкания комбинированного типа, установленным для обеспечения защиты параллельной цепи.AFCI ответвления / фидера разрешается использовать для выполнения требований 210.12 (B) до 1 января 2008 г.

  • Обнаруживает дуговые замыкания, которые стандартные автоматические выключатели не могут обнаружить.

  • Предназначен для смягчения воздействия параллельных дуг (линия-земля или линия-нейтраль) путем отключения питания цепи при обнаружении дугового короткого замыкания.

  • Эксклюзивный светодиодный индикатор для обозначения типа поездки.


Прерыватели цепи замыкания на землю (GFCI) типа QPF являются эффективным средством предотвращения серьезного поражения электрическим током. GFCI устанавливаются для защиты участков дома, таких как кухня, ванная комната или прачечная, где электрические приборы или продукты могут контактировать с водой.

GFCI предназначены для защиты от сильного поражения электрическим током или поражения электрическим током в результате замыкания на землю. Замыкания на землю возникают, когда электрический ток в приборе выходит за пределы своего обычного пути, и человеческое тело становится частью пути, по которому может течь электрический ток.

Для защиты оборудования от повреждений заземления GFCI Siemens непрерывно отслеживают разницу в токе между горячим и нейтральным проводниками. Если электричество, идущее в розетку, равно току, возвращающемуся из розетки, GFCI бездействует. Если электричество, идущее в розетку, больше, чем на 5 мА, чем ток, возвращающийся из розетки, GFCI откроет цепь, чтобы остановить поток электричества.

  • Подходит для различных строительных применений: спа, гидромассажные ванны, кухни, ванные комнаты и т. Д.

  • Устойчив к ложным срабатываниям (экранирован для предотвращения радиопомех)

  • Стандартный формат 1 дюйм на полюс с разъемной конструкцией

  • Обеспечивает защиту класса A GFCI


Двухфункциональные автоматические выключатели типа QFGA2 объединяют класс A 5 мА GFCI и комбинированный тип AFCI, обеспечивая защиту как от дугового замыкания, так и от замыканий на землю. Это, наряду с новой функцией самотестирования, делает его первым в своем классе по электробезопасности для домовладельцев.

Национальный электротехнический кодекс 2014 г. (NEC®) теперь требует защиты как от дугового замыкания, так и от замыкания на землю в цепях кухни и прачечной. До выпуска двухфункциональных автоматических выключателей единственным вариантом соблюдения этого правила было соединение автоматического выключателя AFCI с розеткой GFCI. Автоматический выключатель двойного назначения объединяет эти два устройства в одно решение, которое обеспечивает как экономию средств, так и меньшие хлопоты при установке и обслуживании.

  • Быстрая установка — проще установить одно устройство, чем два

  • Экономия затрат — более низкая стоимость, чем покупка GFCI и AFCI

  • Меньшее устройство — объединяет две новейшие технологии в одно маленькое устройство

  • Функция самотестирования в соответствии с требованиями UL 943 с июня 2015 г. — Автономное и непрерывное самотестирование на предмет неисправности устройства.Отключает устройство при взломе защиты

СКАЧАТЬ PDF — Автоматические выключатели Siemens типа QFGA2

Для получения дополнительной информации о автоматических выключателях Siemens для жилых помещений посетите Siemens USA.

Автоматические выключатели и переключатели в литом корпусе FA (F-frame)

График перехода

По мере того, как сроки будут окончательно определены, даты будут уточнены ниже.

Ноябрь 2016 г .: Представление PowerPact B
Июнь 2017 г .: Объявлено об устаревании автоматических выключателей и выключателей в литом корпусе с F-образной рамой
4 квартал 2018 г .:
Доступны корпуса PowerPact B
Автоматические выключатели I-Line с F-образной рамой и выключатели больше не доступны для покупки
Q2 2019: Автоматические выключатели и переключатели для монтажа на F-образной раме больше не доступны для покупки

Поиск замены
PowerPact B 15-125A Автоматические выключатели и переключатели в литом корпусе предназначены для замены Приложения F-frame.PowerPact B отличается увеличенной емкостью, меньшим размером для установки блока, тем же размером, что упрощает модернизацию I-Line, а также гибким набором аксессуаров, вспомогательных устройств и операторов, устанавливаемых на месте.

Инструменты и ресурсы:

Замена выключателя

Замена крепления блока на F-образной раме / установка корпуса
Для вашего текущего крепления блока / корпуса на F-образной раме могут потребоваться незначительные корректировки для соответствия PowerPact B из-за меньший физический размер нового автоматического выключателя.
Доступны две переходные пластины:

  • HJKADAPT: Замена на винтах автоматических выключателей с F-образной рамой
  • LV426995: F-образная рама на B-образную раму PowerPact 9421 Комплект пластины для преобразования поворотной рукоятки

    ПРИМЕЧАНИЕ:
    Это преобразование пластины также требует новый приводной механизм 9421LB7 и один новый комплект вала: 9421LS8 или 9421LS13.

Замена F-образной рамы Установка I-Line
Никакой адаптации не требуется, если у вас есть приложение I-Line.Автоматические выключатели PowerPact B I-Line и F-frame имеют одинаковый размер, что упрощает процесс модернизации.

Поддержка
Schneider Electric стремится поддерживать ваш бизнес. Пожалуйста, свяжитесь с вашим представителем Schneider Electric для получения дополнительной информации или совместной работы над планами модернизации.

Свяжитесь с нами

Типы и функции автоматических выключателей

Панели — жизненно важная часть любого жилого помещения. Здесь вы познакомитесь с компонентами электрического щитка, а также с типами и функциями автоматических выключателей.

Главная панель и вспомогательная панель для жилых помещений

Panelboard монтируется в стене, управляя и распределяя мощность по ответвленным цепям в доме. Субпанели обычно добавляются для распределения энергии и получения локального управления ответвлениями цепей, таких как гаражи и мастерские, а также позволяют установить больше цепей в какой-то момент в будущем.

Основные компоненты заземленного основного щита обслуживания жилого помещения:

  • Устройства максимального тока: автоматические выключатели
  • Проводники: Однофазные — 240 В (2 ножки)
  • Нейтральный провод: белый (идентифицирован) 120 В от N к L1 или N к L2
  • Провод заземления оборудования (EGC), который заканчивается на шине заземления на главной панели управления
  • Провод заземляющего электрода (GEC), который заканчивается на шине заземления на главной панели и заземлении (заземляющие стержни)
  • Перемычка основного заземления: перемычка между шиной заземления и шиной нейтрали

Как работают панели

Однофазный: Кабель или проводники подают напряжение 120/240 В на главную панель и заканчиваются на выводах главного выключателя, которые считаются (2) ножками, обычно обозначаемыми как L1 и L2.Каждая нога измеряет 120 вольт между нейтралью. Главный автоматический выключатель соединяет левую и правую шины, а нейтраль подключается к нейтральной шине с помощью основной перемычки.

Однополюсные автоматические выключатели (120 вольт), а также двухполюсные автоматические выключатели (240 вольт) подключаются между сборными шинами и ответвленными цепями.

Силовые нагрузки: Электрические плиты и сушилки для одежды работают от сети 120/240 В (нейтраль L1, L2 и заземляющий провод оборудования). Проводники параллельной цепи оканчиваются на двойных автоматических выключателях, а нейтраль оканчивается на нейтральной шине.

Нейтраль и эффективный путь заземления

Чтобы эффективно изучить систему в целом, вам необходимо знать разницу между нейтралью и эффективным заземлением.

  • Нейтраль : Нейтраль относится к изолированному проводнику, который предназначен для проведения тока во время нормальной работы, замыкая цепь.
  • Эффективный путь заземления : Состоит из комбинации заземляющего проводника оборудования и скрепленных металлических частей и оборудования.Нетоковедущие металлические части и оборудование в нем обеспечивают путь с низким сопротивлением, устраняющий замыкания на землю.

Функции автоматического выключателя

Автоматические выключатели имеют решающее значение для защиты параллельных цепей от перегрузок, короткого замыкания и замыканий на землю. Автоматические выключатели — это переключатели, которые могут безопасно размыкать цепи и блокироваться в открытом состоянии, когда приходит время обслуживать / устранять неисправности, изолировать ответвленные цепи, электрические устройства и оборудование. Номинальная сила тока прерывателя обычно печатается на ручке или переключателе.Другая информация на этикетке выключателя может включать:

  • Рейтинг HACR (отопление / кондиционирование / охлаждение) — Он может справиться с кратковременным скачком силы тока, который необходим для запуска компрессора или других устройств.
  • SWD (переключатель) — Может использоваться в качестве переключателя для размыкания или изоляции ответвленных цепей.
  • Рейтинг AIC (отключающая способность в амперах) — Может достигать 10 000 ампер, номинальный ток короткого замыкания.
  • Момент затяжки — Для проводов, заканчивающихся на механических наконечниках (обвязка проводов), требуется затяжка, выраженная в фунт-дюймах, а для установки необходима динамометрическая отвертка.

Определение крутящего момента для автоматических выключателей

При подключении проводника к винту для обвязки проводов, например автоматическим выключателям или механическим наконечникам, надлежащая затяжка является требованием Национального электрического кодекса (NEC).

Если спецификация крутящего момента не указана на этикетке автоматического выключателя, возможно, вам придется поискать правильные настройки крутящего момента для конкретной марки / модели автоматического выключателя в документации производителя или на веб-сайте.

Настройки крутящего момента могут отличаться в зависимости от калибра провода, используемого в системе.Некоторые марки / модели автоматических выключателей могут иметь характеристики крутящего момента и другую информацию, выбитую на боковой стороне пластикового корпуса, а не на этикетке спереди.

Типовые автоматические выключатели для жилых помещений

Автоматические выключатели для бытовых нужд и напряжения включают:

  • Однополюсный — 120 В
  • Тандем —120 В (осторожность с многопроволочными параллельными цепями)
  • Двухполюсный — 240 вольт
  • Автоматические выключатели

предназначены для включения / отключения ответвленных цепей на ВКЛ / ВЫКЛ и ОТКЛЮЧЕНИЕ.Они защищают изоляцию параллельной цепи от чрезмерного нагрева из-за перегрузки по току. Эта сила тока в сочетании с температурой окружающей среды, превышающей номинальную мощность автоматического выключателя, а также короткое замыкание и замыкание на землю, проводят высокий ток из-за низкого импеданса обратного пути к источнику (эффективный путь заземления) ОТКЛЮЧАЮТ автоматический выключатель.

  • A Однополюсный автоматический выключатель предназначен для защиты 120-вольтовых осветительных нагрузок, таких как розетки, прачечная, кухня, жилые комнаты и т. Д.
  • A Двухполюсный автоматический выключатель предназначен для защиты оборудования с напряжением 240 В, такого как электрические плиты, электрические сушилки, электрические резервуары для горячей воды.
  • Тандемные выключатели используют два однополюсных выключателя, размещенных в одном однополюсном пространстве. Это помогает сэкономить место на щитке и оставить достаточно места для добавления других ответвлений в какой-то момент.

Как работают автоматические выключатели

Автоматические выключатели

предназначены для выдерживания кратковременного броска тока, превышающего номинальную мощность, что позволяет двигателям вентиляторов или компрессорам работать.Время, в течение которого они могут допустить этот бросок тока, в сравнении с фактической допустимой силой тока, с которой они могут работать в течение этого времени, выражается на кривой «Время / сила тока» или «Срабатывание».

В зависимости от конструкции и назначения выключателя, он может выдерживать в 15 раз большую нагрузку и может отключиться от миллисекунд до 2-3 секунд позже.

У выключателей три позиции:

  • На — замкнутая цепь
  • Отключение — обрыв цепи также указывает на проблему
  • Выкл. — обрыв цепи

Общие сведения о прерывателях GFCI, расположенных в Panelboards

GFCI Breaker означает прерыватель цепи защиты от замыкания на землю, который не следует путать с розетками GFCI, функция выключателя GFCI — защита от поражения электрическим током, перегрузок, короткого замыкания и замыканий на землю, а розетки GFCI защищают только от поражения электрическим током.

Защита

GFCI обычно применяется для влажных помещений, таких как кухонные прилавки, ванные комнаты, подвалы, внешние бассейны / спа, а также в приложениях, где вода может присутствовать и приводить к замыканию на землю. Защита GFCI требуется NEC и должна быть легко доступна.

Выключатели

GFCI спроектированы с простой схемной платой внутри, которая контролирует ток и силу тока, которые проходят через прерыватель, и затем срабатывает, если обнаруживает дисбаланс силы тока 5 мА, что может означать замыкание на землю.

Например, ответвленная цепь на 120 В. 10 ампер на проводе ответвленной цепи к нагрузке (оборудованию) и 10 ампер, возвращающихся на нейтральный провод, будут нормальным состоянием. 10 ампер на нагрузку (оборудование) и возврат 9,9 ампер указывают на замыкание на землю 100 мА, которого достаточно, чтобы вызвать серьезное или даже смертельное поражение электрическим током. В этот момент печатная плата обнаружит эту разницу> 5 мА и быстро отключит прерыватель GFCI.

Выключатели GFCI будут иметь маркировку с указанием допустимой нагрузки в амперах для службы HACR, но также будут иметь четкую маркировку с их функцией GFCI.Они оснащены кнопкой тестирования и цветным индикатором, который показывает, когда произошло замыкание на землю.

Проводники параллельной цепи выключателей

GFCI заканчиваются на соответствующей клемме нагрузки, а нейтральный провод заканчивается на клемме нейтрали выключателя GFCI, а не идет прямо к нейтральной шине. Отдельный пигтейл от прерывателя GFCI будет подключен к нейтральной шине. Узнайте больше о автоматическом выключателе и розетке GFCI: как защитить себя от поражения электрическим током в нашем блоге.

Прерыватели AFCI

Автоматические выключатели

Arc Fault Circuit Interrupter (AFCI) имеют печатную плату, которая может отличить обычную мгновенную дугу, такую ​​как срабатывание выключателя света, от продолжительного события, вызывающего дугу. Функция выключателя AFCI заключается в обнаружении этих опасных состояний дуги и отключении, предотвращая электрический пожар.

Дуга (параллельная дуга) может возникнуть, если изоляция изношена или защемлена, что позволяет току проходить между горячим и нейтральным проводами. Это также может произойти, если гвоздь или шуруп для гипсокартона пробил провод или повредил внутренний провод внутри электрического шнура.Из-за опасности, создаваемой электрическими дугами, были приняты следующие меры:

  • Защита от AFCI теперь требуется NEC (ответвительные цепи на 15 и 20 А) для жилых комнат, комнат отдыха, офисов, кухонь, спален, туалетов, прачечных и притонов.
  • Автоматические выключатели AFCI четко обозначены на передней панели и оснащены кнопкой тестирования, а также индикатором Arc Fault, который показывает, когда было обнаружено дуговое замыкание.
  • Маркировка автоматических выключателей
  • AFCI должна включать рейтинг HACR, номинальную силу тока и AIC (рейтинг короткого замыкания).

CAFCI, выключатели двойного назначения и розетки AFCI

В то время как автоматические выключатели AFCI могут обнаруживать дугу, переходящую от линии к нейтрали (параллельной), автоматические выключатели CAFCI предназначены для обнаружения дуги Line1-Line1 (последовательной), например, внутри поврежденного электрического шнура устройства. Прерыватели CAFCI помечены как комбинированные AFCI и не обеспечивают вместе защиту AFCI и GFCI. Это потребовало бы прерывателя двойного назначения.

Двухфункциональные выключатели — это довольно новая инновация, предлагающая наилучший возможный пакет защиты.Они помечены как для AFCI, так и для GFCI и имеют отдельные цветные индикаторы, которые загораются при возникновении дугового замыкания или замыкания на землю.

Кроме того, NEC требует наличия автоматических выключателей AFCI, если доступ к устройству затруднен.

Меры безопасности при блокировке / маркировке

Автоматические выключатели как изолирующее устройство

Lockout / Tagout — это способ предотвратить подачу питания в систему путем размещения устройства блокировки / навесного замка и предупреждающей бирки на изолирующем устройстве системы.Часто при выполнении блокировки в ответвленной цепи к электрическому устройству электрооборудование получает питание от автоматического выключателя. Узнайте все, что вам нужно знать о процедурах блокировки тегов, в нашем недавнем сообщении в блоге.

Каждый технический специалист отвечает за установку собственного замка и бирки на оборудование для обеспечения «нулевого энергопотребления». Один ключ от замка!

Оттуда:

  • Квалифицированный специалист (OSHA / NFPA70E) выполняет проверку и изоляцию под напряжением.
  • Тег блокировки подписан лицом, установившим его и работающим в системе.
  • Иногда система может быть заблокирована более чем одним человеком. Для обеспечения безопасности каждый человек должен снять свою блокировку / маркировку, прежде чем система снова будет включена.
  • Накладки для блокировки могут иметь до шести отверстий для отдельных навесных замков; кабели можно использовать для расширения блокировки до отдельного автоматического выключателя.

Эта разбивка щитовых щитов и различных типов автоматических выключателей, их конструкция и функции, а также основные процедуры безопасности для электрических систем являются отличным началом при входе в отрасль.Чтобы получить более подробное представление об этих областях, ознакомьтесь с каталогом курсов SkillMill ™ Electrical.

Чад Суси

Interplay Learning Электротехнический эксперт

Чад — специалист по электрике Interplay и старший электрик. На протяжении всей своей карьеры Чад продвигался в качестве специалиста-электрика, с самого начала занимаясь ремонтом / электромонтажом домов на предприятиях обеспечения качества / вводом в эксплуатацию, попутно оттачивая свои навыки во всех аспектах торговли электроэнергией. Он перешел в свою карьеру через жилые, коммерческие и промышленные объекты, а в 2012 году расширил свою миссию, став инструктором по электрике на протяжении всей жизни.Он продолжил этот путь в качестве разработчика онлайн-курсов и твердо привержен принципам электробезопасности и здравым теориям обучения взрослых.

Бюллетень для инвесторов: меры по устранению волатильности рынка

Управление по обучению и защите инвесторов SEC выпускает этот Бюллетень для инвесторов, чтобы проинформировать инвесторов о мерах безопасности, одобренных SEC для устранения волатильности рынка на фондовых рынках США.

Механизм увеличения и уменьшения лимита — 31 мая 2012 года Комиссия по ценным бумагам и биржам утвердила механизм «Ограничение вверх-вниз» для устранения волатильности рынка путем предотвращения сделок с котируемыми долевыми ценными бумагами, когда они вызваны крупными внезапными изменениями цен у отдельных склад.Механизм ограничения лимита вверх-вниз предназначен для предотвращения сделок с отдельными ценными бумагами за пределами указанного ценового диапазона. Этот ценовой диапазон устанавливается на процентном уровне выше и ниже средней цены акции за непосредственно предшествующий пятиминутный торговый период. Эти диапазоны ценовых пределов составляют 5%, 10%, 20% или меньше 0,15 или 75%, в зависимости от цены акции. Кроме того, эти ценовые диапазоны удваиваются в период открытия и закрытия торгового дня. Если цена акции не вернется в ценовые диапазоны естественным образом в течение 15 секунд, будет установлена ​​пятиминутная торговая пауза.

Пересмотренные рыночные выключатели — Биржи ценных бумаг и фьючерсов имеют процедуры для скоординированных остановок межрыночных торгов, если серьезное снижение рыночных цен достигает уровней, которые могут истощить рыночную ликвидность. Эти процедуры, известные как общерыночные автоматические выключатели, могут временно остановить торговлю или, в экстремальных обстоятельствах, закрыть рынки до обычного закрытия торговой сессии.

Автоматические выключатели для всего рынка обеспечивают остановку межрыночных торгов во время серьезного рыночного спада, измеряемого однодневным снижением индекса S&P 500.Остановка межрыночной торговли может быть инициирована при трех порогах прерывания цепи — 7% (уровень 1), 13% (уровень 2) и 20% (уровень 3). Эти триггеры устанавливаются рынками на уровне пунктов, которые рассчитываются ежедневно на основе цены закрытия предыдущего дня по индексу S&P 500.

Падение рынка, при котором срабатывает автоматический выключатель уровня 1 или уровня 2 до 15:25. остановит торговлю на всем рынке на 15 минут, в то время как аналогичное снижение рынка произойдет «в 15:25 или позже». не остановит торговлю на рынке.Падение рынка, которое вызывает срабатывание выключателя уровня 3 в любое время в течение торгового дня, остановит торговлю на рынке на оставшуюся часть торгового дня.

Для получения дополнительной информации о механизме ограничения вверх-вниз и автоматических выключателях, действующих на рынке, пожалуйста, прочтите следующее:

План национальной рыночной системы — волатильность рынка

План национальной рыночной системы — Волатильность рынка — Поправка 2

План национальной рыночной системы — Волатильность рынка — Поправка 3

Распоряжение об утверждении автоматических выключателей для всего рынка

Обзор электрических схем

— Ответы № 2

Обзор электрических цепей

Переход к:

Главная страница сеанса обзора — Список тем

Electric Circuits — Главная || Версия для печати || Вопросы со ссылками

Ответы на вопросы: Все || # 1-7 || # 8-51 || # 52-59 || # 60-72



Часть B: множественный выбор

8.Если бы электрическая цепь была аналогична аквапарку, то аккумулятор был бы аналогичен ____.

а. трубы, по которым вода проходит через водяной контур

г. насос, который подает энергию для перемещения воды с земли на высоту

г. люди, которые текут с верха водного аттракциона на нижний водный аттракцион

г. скорость, с которой вода закачивается на горку

e. изменение потенциальной энергии гонщиков

ф.верх водной горки

г. дно водной горки

ч. длинные очереди в парке

и. скорость, с которой движутся гонщики при скольжении сверху вниз по траектории

Ответ: B

Водный аттракцион в аквапарке аналогичен электрическому контуру. Во-первых, есть сущность, которая течет — вода течет в аквапарке и (условно) + течет заряд в электрической цепи.В каждом случае текучая среда самопроизвольно течет из места с высокой энергией в место с низкой энергией. Поток идет по трубам (или горкам) в аквапарке и по проводам в электрической цепи. Если трубы или провода порваны, непрерывный поток жидкости через контур невозможен. Для установления цепи требуется полный цикл.

Этот поток жидкости — будь то вода или заряд — возможен, когда создается разница давлений между двумя точками в контуре .В аквапарке перепад давления — это разница напора воды, создаваемая двумя локациями на разной высоте. Вода самопроизвольно течет из мест с высоким давлением (большая высота) в места с низким давлением (низкая высота). В электрической цепи разность электрических потенциалов между двумя выводами батареи или источника энергии обеспечивает электрическое давление, которое оказывает давление на заряд, чтобы переместить их из места высокого давления (высокого электрического потенциала) в место низкого давления (низкий электрический потенциал). потенциал).

Энергия требуется для перемещения жидкости вверх по склону . В аквапарке водяной насос используется для работы с водой, чтобы поднять ее с небольшой высоты обратно на большую. Водяной насос не подает воду; вода, которая уже есть в трубах. Напротив, водяной насос подает энергию для перекачивания воды из места с низкой энергией и низким давлением в место с высокой энергией и высоким давлением. В электрической цепи аккумулятор является зарядным насосом, который прокачивает заряд через аккумулятор из места с низким электрическим потенциалом (клемма -) в место с высоким электрическим потенциалом (клемма +).Аккумулятор не подает электрический заряд; заряд уже в проводах. Аккумулятор просто поставляет энергию для работы над зарядом, перекачивая его на в гору .


9. Если бы электрическая цепь была аналогична аквапарку, то положительный полюс батареи был бы аналогичен ____.

а. трубы, по которым вода проходит через водяной контур

г.насос, который подает энергию для перемещения воды с земли на высоту

г. люди, которые текут с верха водного аттракциона на нижний водный аттракцион

г. скорость, с которой вода закачивается на горку

e. изменение потенциальной энергии гонщиков

ф. верх водной горки

г. дно водной горки

ч. длинные очереди в парке

и.скорость, с которой движутся гонщики при скольжении сверху вниз по траектории


Ответ: F

Водный аттракцион в аквапарке аналогичен электрическому контуру. Во-первых, есть сущность, которая течет — вода течет в аквапарке и (условно) + течет заряд в электрической цепи. В каждом случае текучая среда самопроизвольно течет из места с высокой энергией в место с низкой энергией.Поток идет по трубам (или горкам) в аквапарке и по проводам в электрической цепи. Если трубы или провода порваны, непрерывный поток жидкости через контур невозможен. Для установления цепи требуется полный цикл.

Этот поток жидкости — будь то вода или заряд — возможен, когда создается разница давлений между двумя точками в контуре . В аквапарке перепад давления — это разница напора воды, создаваемая двумя локациями на разной высоте.Вода самопроизвольно течет из мест с высоким давлением (большая высота) в места с низким давлением (низкая высота). В электрической цепи разность электрических потенциалов между двумя выводами батареи или источника энергии обеспечивает электрическое давление, которое оказывает давление на заряд, чтобы переместить их из места высокого давления (высокого электрического потенциала) в место низкого давления (низкий электрический потенциал). потенциал).

Энергия требуется для перемещения жидкости вверх по склону .В аквапарке водяной насос используется для работы с водой, чтобы поднять ее с небольшой высоты обратно на большую. Водяной насос не подает воду; вода, которая уже есть в трубах. Напротив, водяной насос подает энергию для перекачивания воды из места с низкой энергией и низким давлением в место с высокой энергией и высоким давлением. В электрической цепи аккумулятор является зарядным насосом, который прокачивает заряд через аккумулятор из места с низким электрическим потенциалом (клемма -) в место с высоким электрическим потенциалом (клемма +).Аккумулятор не подает электрический заряд; заряд уже в проводах. Аккумулятор просто поставляет энергию для работы над зарядом, перекачивая его на в гору .


10. Если бы электрическая цепь была аналогична аквапарку, то электрический ток был бы аналогичен ____.

а. трубы, по которым вода проходит через водяной контур

г.насос, который подает энергию для перемещения воды с земли на высоту

г. люди, которые текут с верха водного аттракциона на нижний водный аттракцион

г. скорость, с которой вода закачивается на горку

e. изменение потенциальной энергии гонщиков

ф. верх водной горки

г. дно водной горки

ч. длинные очереди в парке

и.скорость, с которой движутся гонщики при скольжении сверху вниз по трассе

Ответ: D

Течение воды в аквапарке аналогично потоку заряда в электрической цепи. Скорость, с которой заряд проходит через точку в цепи, измеряемая в кулонах заряда в секунду (или некотором сопоставимом наборе единиц), называется током. В нашей аналогии текучая среда, которая течет, — это вода, а скорость, с которой жидкость проходит через любую заданную точку, — это течение.

11. Потенциальная энергия единицы заряда в любом заданном месте называется электрической ___.

а. текущий

г. сопротивление

г. потенциал

г. мощность

Ответ: C

Это определение электрического потенциала — понятие, которое вы должны усвоить.

[# 8 | # 9 | # 10 | # 11 | # 12 | # 13 | # 14 | # 15 | # 16 | # 17 | # 18 | # 19 | # 20 | # 21 | # 22 | # 23 | # 24 | # 25 | # 26 | # 27 | # 28 | # 29 | # 30 | # 31 | # 32 | # 33 | # 34 | # 35 | # 36 | # 37 | # 38 | # 39 | # 40 | # 41 | # 42 | # 43 | # 44 | # 45 | # 46 | # 47 | # 48 | # 49 | # 50 | # 51]

12. Один ампер — это величина тока, которая существует, когда ____ протекает через определенную точку в проводнике в ____.

а.один ватт; одна секунда

г. один джоуль; один час

г. один электрон; одна секунда

г. один электрон; один час

e. один вольт; одна секунда

ф. один вольт; один час

г.один кулон; одна секунда

ч. один кулон; один час

Ответ: G

Ампер — единица измерения электрического тока. Электрический ток определяется как скорость, с которой заряд проходит через точку в цепи, измеряемую в стандартных единицах кулонов заряда в секунду.

[# 8 | # 9 | # 10 | # 11 | # 12 | # 13 | # 14 | # 15 | # 16 | # 17 | # 18 | # 19 | # 20 | # 21 | # 22 | # 23 | # 24 | # 25 | # 26 | # 27 | # 28 | # 29 | # 30 | # 31 | # 32 | # 33 | # 34 | # 35 | # 36 | # 37 | # 38 | # 39 | # 40 | # 41 | # 42 | # 43 | # 44 | # 45 | # 46 | # 47 | # 48 | # 49 | # 50 | # 51]

13.Если 6 кулонов заряда проходят мимо точки «А» в контуре за 4 секунды, то ____ кулонов заряда проходят мимо точки «А» за 8 секунд.

а. 0,67

г. 1,5

г. 2

г. 3

e. 4

ф.6

г. 8

ч. 12

и. 24

Ответ: H

Ток (I) — это количество заряда, протекающего через точку (Q) за заданный промежуток времени (t). То есть I = Q / t. Таким образом, в этом случае ток в точке A равен (6 C) / (4 с) или 1.5 ампер. Таким образом, отношение Q / t составляет 1,5 независимо от времени. Решите уравнение

1,5 Кл / с = Q / (8 с)

для Q, чтобы получить ответ.

[# 8 | # 9 | # 10 | # 11 | # 12 | # 13 | # 14 | # 15 | # 16 | # 17 | # 18 | # 19 | # 20 | # 21 | # 22 | # 23 | # 24 | # 25 | # 26 | # 27 | # 28 | # 29 | # 30 | # 31 | # 32 | # 33 | # 34 | # 35 | # 36 | # 37 | # 38 | # 39 | # 40 | # 41 | # 42 | # 43 | # 44 | # 45 | # 46 | # 47 | # 48 | # 49 | # 50 | # 51]

14.В какой из следующих ситуаций загорится лампочка? Перечислите все подходящие варианты.

Ответ: DF

Для установления цепи должен быть замкнутый проводящий контур от положительной клеммы к отрицательной. Это будет означать, что цепи D, E и F будут цепями. Но чтобы лампочка загорелась, ее необходимо включить в электрическую цепь. Итак, в E лампочка не загорается, поскольку петля не проходит в лампочку и не проходит сквозь нее; заряд будет просто вытекать из + клеммы батареи и прямо обратно в отрицательную клемму батареи.

Для вопросов № 15- № 17:

Простая схема, содержащая аккумулятор и лампочку, показана на схеме справа. Используйте эту диаграмму, чтобы ответить на несколько следующих вопросов.

15. Ток через батарею ___.

а. больше, чем через лампочку

г.меньше, чем через лампочку

г. то же, что и через лампочку

г. больше, чем через каждый провод

e. меньше, чем через каждый провод

Ответ: C

Начисление — это сохраняемая величина; он никогда не приобретается и не теряется.В электрической цепи заряд, присутствующий в проводах и проводящих элементах, — это то, что движется по цепи. Этот заряд заключен в провода и не может выйти (при условии, что в цепи нет неисправности). По мере того, как заряд течет, он не накапливается в данном месте. И заряд не израсходован, а как бы расходное количество. При этом заряд не трансформируется в другой тип сущности. Учитывая все эти рассуждения, можно было бы заключить, что ток в одном месте в электрической цепи такой же, как ток в любом другом месте в электрической цепи.

16. Заряд, протекающий по этой цепи, имеет наибольшее напряжение в ____. Выберите один лучший ответ.

а. + клемма аккумулятора

г. — клемма аккумулятора

г. непосредственно перед входом в лампочку

г. сразу после выхода из лампочки

e. … ерунда! Энергия заряда одинакова во всем контуре.

Ответ: A

Клемма «+» батареи является высокоэнергетической клеммой батареи.

17. Роль или назначение батареи в этой цепи — ____. Выберите три.

а. подавать электрический заряд, чтобы мог существовать ток

г. подавать энергию к заряду

г.переместите заряд с — на + вывод аккумуляторной батареи

г. преобразовать энергию из электрической энергии в световую

e. установите разность электрических потенциалов между клеммами + и —

ф. восполнить потерянный в лампочке заряд

г. Обеспечьте сопротивление потоку заряда, чтобы лампочка могла нагреваться

Ответ: до н.э.

Чтобы установить электрическую цепь, заряд должен быть переведен с низкой энергии на высокую.При достижении высокой энергии заряд самопроизвольно течет через проводящие провода и другие проводящие элементы схемы назад вниз к клемме с низким энергопотреблением. Роль батареи заключается в обеспечении энергией, необходимой для переноса заряда с клеммы — на клемму + батареи. Помещая большое количество одного и того же заряда в одном месте, устанавливается электрическое давление или разность потенциалов, заставляя одинаковые заряды перемещаться из этого места в место противоположного заряда (клемма -).


18. Аккумулятор на 12 В будет обеспечивать ___. Перечислите все подходящие варианты.

а. 3 кулоны заряда с 4 джоулями энергии

г. 4 кулоны заряда с 3 джоулями энергии

г. 12 кулонов заряда с 1 Джоулем энергии

г. 1 кулон заряда с энергией 12 джоулей

e. 0,5 кулонов заряда с энергией 24 джоулей

ф.24 кулоны заряда с 2 джоулями энергии

Ответ: D

Электрический потенциал (или напряжение) определяется как электрическая потенциальная энергия на заряд. Это джоули энергии на кулон заряда, которыми обладает некоторое количество заряда в некотором месте электрической цепи. Аккумулятор на 12 В перемещает некоторое количество заряда с клеммы — на клемму +, передавая энергию заряда. Каждый кулон заряда потреблял бы 12 Джоулей энергии.Соотношение энергия / заряд будет 12 Дж / Кл.

19. Заряды, протекающие по проводам в вашем доме ____.

а. хранятся в торговых точках у вас дома

г. создаются при включении устройства

г. происходят в энергетической компании

г. берут начало в проводах между вашим домом и энергокомпанией

e. уже есть в проводах у вас дома

Ответ: E

Этот вопрос направлен против распространенного заблуждения об электрических цепях.Заблуждение предполагает, что роль электрической розетки, аккумулятора или энергокомпании заключается в обеспечении заряда, необходимого для передвижения по дому. Но энергетическая компания является только источником энергии, необходимой для приведения заряда в движение, путем установления разности электрических потенциалов. Сам заряд присутствует в проводах и токопроводящих элементах вашего дома в виде мобильных электронов.

20.Примерно сколько времени потребуется электрону, чтобы пройти от аккумуляторной батареи автомобиля до фары и обратно (полный цикл)?

а. секунды

г. часы

г.

лет

г. одна миллионная секунды

e. одна десятая секунды

Ответ: B

Электрический заряд, проходящий по электрической цепи, движется довольно медленно.Довольно удивительно для многих, что расстояние, пройденное за единицу времени, составляет порядка 1 метра в час.

21. Представленная справа электрическая схема состоит из аккумулятора и трех одинаковых лампочек. Какие из следующих утверждений относительно этой схемы верны? Перечислите все подходящие варианты.

а. Ток через точку X будет больше, чем через точку Z.

г. Ток через точку Z будет больше, чем через точку Y.

г. Ток будет одинаковым через точки X, Y и Z.

г. Ток через точку X будет больше, чем через точку Y.

e. Ток через точку Y будет больше, чем через точку X.

Ответ: C

Как обсуждалось в вопросе № 15 выше, ток в электрической цепи везде одинаков. Таким образом, ток в этих трех местах одинаков.

22. Представленная справа электрическая схема состоит из аккумулятора и трех одинаковых лампочек. Какие из следующих утверждений относительно этой схемы верны? Перечислите все подходящие варианты.

а. Разность электрических потенциалов между X и Y больше, чем между Y и Z.

г. Разность электрических потенциалов между X и Z больше, чем между Y и W.

г.Разность электрических потенциалов между X и Y такая же, как между Y и Z.

г. Разность электрических потенциалов между X и Z такая же, как между Y и W.

e. Разность электрических потенциалов между Y и W больше, чем между X и Y.

Ответ: DE

Разность электрических потенциалов на лампочке (или на любом резисторе) в электрической цепи — это просто произведение тока в этой лампочке на ее сопротивление.Каждая лампочка имеет одинаковое сопротивление (поскольку они идентичны) и одинаковый ток (поскольку ток везде одинаковый). Таким образом, разность электрических потенциалов на каждой лампочке одинакова. И падение потенциала на любых двух последовательных лампочках одинаково. И падение потенциала на двух лампах будет больше, чем на одной лампочке.

23. Электрическая схема, показанная справа, состоит из аккумулятора и трех одинаковых лампочек.Какие из следующих утверждений относительно этой схемы верны? Перечислите все подходящие варианты.

а. Обычный ток направляется по внешней цепи от точки X к Y, от Z к W.

г. Обычный ток направляется через внешнюю цепь от точки W к Z, к Y к X.

г. Обычный ток направляется по внутренней цепи от точки W к точке X.

г. Обычный ток направляется по внутренней цепи из точки X в точку W.

e. Точка, в которой заряд обладает наименьшим количеством электрической потенциальной энергии, — это точка W.

Ответ: ACE

Батарея называется внутренней схемой. Заряд перемещается по внутренней цепи от клеммы — к клемме + (в направлении от W к Z). Провода и лампочки составляют внешнюю цепь; заряд движется по внешней цепи от клеммы + к клемме — (в направлении от X к Y, от Z к W).

24. Напряжение ____ в электрической цепи.

а. проходит через

г. выражается через

г. постоянно на протяжении

г. скорость, с которой расходы проходят через

Ответ: B

Напряжение или электрический потенциал не движутся.Таким образом, варианты A и D не являются ответами, поскольку предполагают изменение напряжения. И напряжение или электрический потенциал заряда не является чем-то постоянным во всей цепи, как предполагает вариант C.

Напряжение или электрический потенциал — это мера того, насколько заряжено количество заряда в данном месте относительно клеммы -. Часто это выражается как разница между двумя точками. Возможно, вы обратили внимание на эту формулировку «потенциал через …» в нескольких ответах в этом обзоре.

25. Два или более из следующих слов и фраз означают одно и то же. Определите их, перечислив их буквы.

а. Напряжение

г. Мощность

г. Разница электрических потенциалов

г. Скорость движения платежей

e.Электрическое давление

ф. Энергия

Ответ: ACE

Напряжение или разность электрических потенциалов являются синонимами. Напряжение не является синонимом энергии. В то время как напряжение (или разность электрических потенциалов) является мерой того, насколько заряжено количество заряда в данном месте, напряжение выражается как энергия на заряд (а не просто как энергия).По аналогии между аквапарком и электрической цепью, напряжение — это мера количества электрического давления, оказываемого на заряд, заставляя его перемещаться из одного места в другое.

Мощность — это синоним мощности. Ток является синонимом скорости, с которой течет заряд.


26. Аккумулятор высокого напряжения может ____.

а. много работать над каждым зарядом, с которым он сталкивается

г.выполнять много работы в течение срока службы

г. протолкнуть много заряда через цепь

г. длиться долго

Ответ: A

Напряжение относится к энергии / заряду. Батарея, рассчитанная на высокое напряжение, может выполнять большую работу на каждый кулон заряда, с которым она сталкивается. В зависимости от размера батареи он может или не сможет выполнять большую работу в течение своего срока службы.



27. Что из перечисленного происходит при перезарядке аккумуляторной батареи?

а. Батарея, мощность которой разряжена, восстанавливается.

г. Батарея, у которой закончился ток, возвращается в нее.

г. Батарея, которая разрядилась, возвращается к ней.

г. Батарея, в которой закончились химические реактивы, подверглась химическому преобразованию.

Ответ: D

Батареи выполняют свои задачи по энергоснабжению, используя энергию экзотермической окислительно-восстановительной реакции для работы при зарядке в электрической цепи. Когда батарея больше не работает, ее реагенты расходуются до такой степени, что электрический потенциал, который реагенты способны производить, невелик по сравнению с общим сопротивлением цепи. В такой момент времени способность индуцировать ток ограничена до такой степени, что элементы внешней цепи больше не работают.

Не все батареи можно перезаряжать. Те, которые являются перезаряжаемыми, могут превращать продукты обратно в реагенты. Зарядное устройство использует электрическую энергию из розетки, чтобы обратить вспять ранее экзотермическую реакцию, превращая ее продукты обратно в реагенты.


28. Птицы могут спокойно стоять на высоковольтных линиях электропередачи. Это потому что ____.

а.они имеют низкий потенциал по отношению к земле.

г. они не оказывают сопротивления току.

г. они всегда выбирают неиспользуемые линии электропередач.

г. разность потенциалов между их ногами мала.

e. они идеальные изоляторы.

ф. они прекрасные дирижеры.

Ответ: D

Чтобы заряд протекал между двумя точками, между этими двумя точками должна быть установлена ​​разность электрических потенциалов.Если птица ставит левую ногу на линию электропередачи, а правую ногу на расстоянии нескольких сантиметров от той же линии электропередачи, то разницы потенциалов между его двумя ногами практически нет. Без разности электрических потенциалов заряд не будет проходить через птицу, и птица будет в безопасности.

29. Когда лампочка в вашей лампе больше не работает, это потому, что в лампочке _____.

а. заканчивается энергия и больше не может качать заряд

г.нет напряжения и необходимо зарядить

г. закончились электроны и поэтому нет больше тока

г. сгорел все ватты и больше не светит

e. сработал автоматический выключатель и должен быть закреплен на блоке предохранителей

ф. обрыв нити накала, что привело к обрыву цепи

г. … ерунда! Лампочка в порядке; вашей семье просто нужно полностью оплатить счет за электроэнергию.

Ответ: F

Самая частая причина неспособности лампочки зажигать — обрыв нити накала.Спиральная вольфрамовая проволока протягивается между двумя вертикальными опорами. Если потревожить в горячем состоянии или из-за чрезмерного износа, металлический вольфрам может сломаться и оставить зазор между двумя вертикальными опорами. Этот разрыв представляет собой разрыв цепи; замкнутый проводящий контур больше не устанавливается, и заряд не течет.



30. В цепи вашего фонаря нужна батарейка, чтобы ____.

а.заряд предоставляется на провода

г. энергия света уравновешивается аккумулятором

г. возможна экзотермическая реакция, создающая свет

г. в цепи

поддерживается разность электрических потенциалов.

e. электроны подаются, чтобы зажечь лампочку

Ответ: D

Одна из функций батареи — просто установить разницу в электрическом потенциале между двумя ее выводами.Заряд с высоким потенциалом будет проходить через внешнюю цепь в место с низким потенциалом.


31. При включении освещения в помещении они сразу загораются. Лучше всего это объясняется тем, что ____.

а. электроны очень быстро перемещаются от переключателя к нити накала лампочки

г. электроны, присутствующие повсюду в цепи, движутся мгновенно

Ответ: B

Электроны очень медленно перемещаются из одного места в другое.Но как только цепь замыкается, они сразу начинают движение. Пока электроны движутся примерно на метр или за час, фактический сигнал, который говорит им начать движение, может двигаться со скоростью света. Таким образом, как только переключатель включен, по цепи циркулирует сигнал, чтобы электроны маршировали . Электроны присутствуют в нити накала цепи.


32. Скорость дрейфа подвижных носителей заряда в электрических цепях ____.

а. очень быстро; меньше, но очень близко к скорости света

г. быстрый; быстрее, чем самая быстрая машина, но далеко не скорость света

г. медленный; медленнее Майкла Джексона пробегает 220-метровую

г. очень медленно; медленнее улитки

Ответ: D

Скорость дрейфа — это расстояние, на которое заряд перемещается за единицу времени.Это значение очень мало, поскольку электроны движутся очень и очень медленно. Двигаясь со скоростью около 1 метра в час, они буквально медленнее, чем улитка.


33. Предположим, что ток в типовой цепи (постоянный ток) велик. Это показатель того, что ____.

а. мобильные носители заряда движутся очень быстро

г. большое количество мобильных носителей заряда продвигается в секунду

г.и a, и b верны

Ответ: B

Ток (скорость, с которой заряд движется мимо точки в цепи) и скорость дрейфа (расстояние, на которое заряд проходит за секунду) не следует путать (и часто это так). Если ток большой, можно быть уверенным только в одном: много зарядов перемещается вперед через точку в цепи каждую секунду.


34.Какие из следующих утверждений представляют правильные эквиваленты единиц измерения? Перечислите все подходящие варианты.

а. 1 Ампер = 1 Кулон в секунду

г. 1 Джоуль = 1 В / кулон

г. 1 Ватт = 1 Джоуль • секунда

г. 1 Вт = 1 В • Кулон в секунду

e.1 Джоуль / Ом = 1 Ампер • Кулон

ф. 1 Джоуль • Ом = 1 Вольт 2 • секунда

Ответ: ADEF

Этот вопрос требует знания как единиц измерения электрических величин, так и уравнений, связывающих эти величины.

При выборе a, ампер — это единица измерения тока (I), а кулон в секунду — это единица заряда в единицу времени (Q / t).Это согласуется с уравнением I = Q / t.

При выборе b джоуль — это единица энергии (Э), а вольт / кулон — это единица измерения напряжения на единицу заряда (В / Кв). Поскольку напряжение — это энергия, приходящаяся на заряд, мы ожидаем, что энергия будет эквивалентна напряжению • заряда. Таким образом, неправильно приравнивать единицы энергии к единицам напряжения на заряд.

При выборе c, ватт — это единица мощности (P), а джоуль • секунда — это единица энергии (E), умноженная на единицу времени (t).Но мощность — это энергия / время, а не энергия • время, так что это неправильный эквивалент единиц.

При выборе d ватт — это единица мощности (P). Справа вольт — это единица измерения напряжения (В), а кулон в секунду — это единица измерения тока (I). Итак, поскольку P = I • V, это правильная эквивалентность единиц.

При выборе e джоуль / Ом — это единица энергии на единицу сопротивления (E / R). Ампер • Кулон — это единица измерения тока, умноженная на единицу заряда (I • Q).Таким образом, уравнение предполагает, что E / R = I • Q. Это можно переставить алгебраически, чтобы сказать, что E / Q = I • R. Поскольку напряжение — это энергия, приходящаяся на заряд (E / Q), уравнение можно переписать как V = I • R. Таким образом, это правильная эквивалентность единиц измерения.

При выборе f джоуль • Ом — это единица энергии, умноженная на единицу сопротивления (E • R). Вольт 2 / сек — это единица измерения напряжения 2 , умноженная на единицу времени (В 2 • t). Таким образом, это уравнение предполагает, что E • R = V 2 • t.Это можно переставить алгебраически, чтобы сказать, что E / t = V 2 / R. Правая часть уравнения эквивалентна мощности, поэтому уравнение можно переписать как P = V 2 / R. правильный способ записи уравнения мощности, эквивалентность данной единицы верна.

35. На какой из следующих схем изображены последовательно включенные резисторы? Перечислите все подходящие варианты.

Ответ: B

A и C представляют собой параллельные соединения, как показано разветвлением, которое происходит до и после резисторов.В варианте B нет разветвления, поэтому резисторы последовательно соединяются.

Вопросы № 36- № 39:

На схеме справа показаны два идентичных резистора — R 1 и R 2 , включенные в цепь с 12-вольтовой батареей. Используйте эту диаграмму, чтобы ответить на несколько следующих вопросов.

36. Эти два резистора соединены в ____.

а.серия

г. параллельно

г. ни

Ответ: A

Можно начать с плюсовой клеммы аккумулятора и начать водить пальцем по проводу. Если когда-либо существует точка, в которой провод подходит к стыку и разветвляется в двух или более направлениях, тогда схема имеет параллельное соединение.В противном случае это последовательная цепь. На этой диаграмме нет разветвлений. Таким образом, это последовательная схема.

37. Разность электрических потенциалов (падение напряжения) на каждом резисторе составляет ___ Вольт.

а. 6

г. 12

г. 24

г…. ерунда!. Разность электрических потенциалов зависит от фактического сопротивления резисторов

.

Ответ: A

Заряд получает увеличение электрического потенциала на 12 вольт при перемещении по внутренней цепи (аккумулятор). Таким образом, когда заряд покидает батарею и проходит через внешнюю цепь, общее падение электрического потенциала должно составлять 12 вольт.Это падение напряжения происходит в два этапа, когда заряд проходит через каждый из резисторов. Заряд потеряет 6 вольт на первом резисторе и 6 вольт на втором резисторе, вернув его к нулю к тому времени, когда он вернется на клемму — батареи. Диаграмма потенциальных возможностей справа является визуальным средством представления этой важной концепции.


38. Если третий резистор (R 3 ), идентичный двум другим, добавлен последовательно с первыми двумя, то общее сопротивление будет ____, а общий ток будет ____.

а. прибавка, прибавка

г. уменьшение, уменьшение

г. увеличение, уменьшение

г. уменьшение, увеличение

e. увеличиваются, остаются прежними

ф. уменьшаются, остаются прежними

г.оставить прежним, увеличить

ч. остаются прежними, уменьшаются

и. остаются прежними, остаются прежними

Ответ: C

Увеличение количества резисторов в последовательной цепи приведет к увеличению общего сопротивления этой цепи и уменьшению тока.(Обратное верно для параллельной схемы.)


39. Если третий резистор (R 3 ), идентичный двум другим, добавлен последовательно с первыми двумя, то разность электрических потенциалов (падение напряжения) на каждом из трех отдельных резисторов будет ____.

а. увеличить

г.уменьшение

г. остаются прежними

Ответ: B

Используя те же рассуждения, что и в вопросе № 37, мы можем сказать, что заряд приобретает 12 Вольт при прохождении через батарею. Он должен будет потерять эти 12 вольт в три этапа при прохождении через внешнюю цепь. Поскольку теперь во внешней цепи есть три падения напряжения вместо двух первоначальных, каждое падение должно быть меньше, чем раньше.Таким образом, на каждом резисторе будет падение напряжения на 4 В (вместо исходных 6 В).


Вопросы № 40- № 43:

На схеме справа показаны два идентичных резистора — R 1 и R 2 , включенные в цепь с 12-вольтовой батареей. Используйте эту диаграмму, чтобы ответить на несколько следующих вопросов.

40. Эти два резистора соединены в ____.

а. серия

г. параллельно

г. ни

Ответ: B

Можно начать с плюсовой клеммы аккумулятора и начать водить пальцем по проводу. Если когда-либо существует точка, в которой провод подходит к стыку и разветвляется в двух или более направлениях, тогда схема имеет параллельное соединение.В противном случае это последовательная цепь. На этой диаграмме есть некоторые разветвления. Когда заряд достигает точки разветвления, он проходит либо через резистор в левой ветви (R 1 ), либо через резистор в правой ветви (R 2 ). Таким образом, это параллельная схема.


41. Разность электрических потенциалов (падение напряжения) на каждом резисторе составляет ___ Вольт.

а.6

г. 12

г. 24

г. … ерунда!. Разность электрических потенциалов зависит от фактического сопротивления резисторов

.

Ответ: B

Заряд получает увеличение электрического потенциала на 12 вольт при перемещении по внутренней цепи (аккумулятор).Таким образом, когда заряд покидает батарею и проходит через внешнюю цепь, общее падение электрического потенциала должно составлять 12 вольт. Это падение напряжения происходит за один шаг, поскольку заряд проходит только через один резистор на обратном пути к батарее. Таким образом, поскольку для заряда выбирается либо левая, либо правая ветвь (но не обе), любая ветвь должна обеспечивать падение напряжения на 12 В. В параллельных цепях разность электрических потенциалов на батарее равна разности электрических потенциалов на любой ветви.Диаграмма потенциальных возможностей справа является визуальным средством представления этой важной концепции.


42. Если третий резистор (R 3 ), идентичный двум другим, добавить параллельно с первыми двумя, то общее сопротивление будет ____, а общий ток будет ____.

а. прибавка, прибавка

г.уменьшение, уменьшение

г. увеличение, уменьшение

г. уменьшение, увеличение

e. увеличиваются, остаются прежними

ф. уменьшаются, остаются прежними

г. оставить прежним, увеличить

ч.остаются прежними, уменьшаются

и. остаются прежними, остаются прежними

Ответ: D

Добавление идентичного резистора в отдельную ветвь обеспечит больше путей, по которым заряд может проходить через петлю цепи. Это было бы эквивалентом добавления еще одной будки на пункте взимания платы на платной дороге параллельно с существующей будкой.Открытие другой полосы движения снизит общее сопротивление и приведет к увеличению скорости потока автомобилей. То же самое происходит с зарядом в параллельных цепях. Больше ответвлений означает меньшее сопротивление и повышенный ток.

43. Если третий резистор (R 3 ), идентичный двум другим, добавить параллельно с первыми двумя, то разность электрических потенциалов (падение напряжения) на каждом из трех отдельных резисторов будет ____.

а. увеличить

г. уменьшение

г. остаются прежними

Ответ: C

Разность электрических потенциалов на любой ветви равна напряжению батареи. Добавление новой ветви может изменить общее сопротивление и общий ток, но не меняет разность электрических потенциалов ни на батарее, ни на ветвях.


[# 8 | # 9 | # 10 | # 11 | # 12 | # 13 | # 14 | # 15 | # 16 | # 17 | # 18 | # 19 | # 20 | # 21 | # 22 | # 23 | # 24 | # 25 | # 26 | # 27 | # 28 | # 29 | # 30 | # 31 | # 32 | # 33 | # 34 | # 35 | # 36 | # 37 | # 38 | # 39 | # 40 | # 41 | # 42 | # 43 | # 44 | # 45 | # 46 | # 47 | # 48 | # 49 | # 50 | # 51]

44. Сопротивление токонесущего провода увеличится на ____. Выберите все, что подходит.

а. длина провода увеличена

г.сечение провода увеличено

г. температура проволоки повышена

г. напряжение на концах провода увеличивается

e. провод ставим все ближе и ближе к + клемме цепи

Ответ: AC

Сопротивление провода увеличивается с увеличением длины и (в меньшей степени) с повышением температуры.Увеличение длины провода увеличивает количество столкновений заряда атома и, следовательно, величину сопротивления. Повышение температуры увеличивает удельное сопротивление материала и, таким образом, увеличивает общее сопротивление.

45. При подключении к розетке на 120 В лампочка потребляет 300 джоулей энергии в течение 5 секунд. Мощность лампочки ____ Вт.

а.0,0167

г. 0,50

г. 2,0

г. 2,50

e. 60

ф. 600

г. 1500

ч. 7200

Ответ: E

Мощность — это просто скорость, с которой энергия подается в цепь или преобразуется в ней.В этом случае мощность — это энергия, потребляемая за раз.

P = (300 Дж) / (5 секунд) = 60 Вт

46. Определенная электрическая цепь содержит аккумулятор, провода и лампочку. Если потенциальная энергия приобретается за счет заряда в месте расположения батареи, тогда заряды теряют потенциальную энергию ____.

а. только в проводах

г. в лампочке только

г. поровну в проводах и лампочке

г.в основном в проводах но немного в лампочке

e. в основном в лампочке, но немного в проводах

ф. никуда

Ответ: E

Charge теряет энергию при прохождении через зоны сопротивления. При последовательном соединении участки с наибольшим сопротивлением преобразуют электрическую энергию в другие формы с большей скоростью. Таким образом, энергия будет потеряна в лампочке и в проводах в гораздо меньшей степени.



47. Электрическая лампочка с высоким сопротивлением и лампочка с низким сопротивлением последовательно подключены к 6-вольтовой батарее. Какая из двух лампочек будет светить ярче всего?

а. У них будет одинаковая яркость.

г. Лампа с низким R будет светиться ярче.

г. Лампа с высоким R будет светиться ярче.

г. Невозможно сделать такой прогноз, поскольку яркость лампы не зависит от сопротивления лампы.

Ответ: C

Поскольку две лампочки включены последовательно, каждая из них испытывает одинаковый ток (i). Мощность будет отдана продуктом i 2 • R. Поскольку i одинаково для каждой лампочки, лампа с наибольшим сопротивлением будет иметь наибольшую мощность. Таким образом, лампочка с высоким R будет преобразовывать электрическую энергию в энергию света с максимальной скоростью и, таким образом, будет светить наиболее ярко.

48.Лампочка с высоким сопротивлением и лампочка с низким сопротивлением подключены параллельно и питаются от 6-вольтовой батареи. Какая из двух лампочек будет светить ярче всего?

а. У них будет одинаковая яркость.

г. Лампа с низким R будет светиться ярче.

г. Лампа с высоким R будет светиться ярче.

г. Невозможно сделать такой прогноз, поскольку яркость лампы не зависит от сопротивления лампы.

Ответ: B

Поскольку две лампочки включены параллельно, каждая из них испытывает одинаковое падение напряжения (В).Мощность будет отдана продуктом i 2 • R. Поскольку V одинаково для каждой лампочки, лампа с наибольшим сопротивлением будет иметь наименьший ток. Ток имеет наибольшее значение при определении мощности лампочки, поскольку в уравнении он возведен в квадрат. Таким образом, лампочка с низким сопротивлением будет иметь наибольший ток и, таким образом, преобразовывать электрическую энергию в энергию света с наибольшей скоростью; он будет сиять наиболее ярко.


49.Три одинаковые лампочки подключены к батарее, как показано справа. Какие настройки можно было бы внести в схему, чтобы увеличить ток, измеряемый в точке X? Включите все, что применимо.

а. увеличить сопротивление одной из лампочек

г. увеличить сопротивление двух лампочек

г. уменьшить сопротивление двух лампочек

г. увеличить напряжение АКБ

e. уменьшить напряжение АКБ

ф.снимаем одну из лампочек

Ответ: CDF

Ток в последовательной цепи (как полный ток, так и ток через отдельные резисторы) напрямую зависит от напряжения батареи и обратно пропорционально полному сопротивлению цепи. Этот ток можно увеличить, увеличив напряжение аккумулятора. Его также можно увеличить, уменьшив общее сопротивление. Удаление лампы уменьшило бы общее сопротивление, а уменьшение сопротивления любой отдельной лампы уменьшило бы общее сопротивление.


50. Три одинаковые лампочки (обозначенные X, Y и Z) подключены к батарее, как показано справа. Какие настройки можно внести в схему ниже, чтобы увеличить ток в точке P? Перечислите все подходящие варианты.

а. увеличить сопротивление одной из лампочек

г. увеличить сопротивление двух лампочек

г. уменьшить сопротивление двух лампочек

г.увеличить напряжение АКБ

e. уменьшить напряжение АКБ

ф. снимаем одну из лампочек

Ответ: CD

Точка P представляет собой место, где можно измерить полный ток этой параллельной цепи. Полный ток будет напрямую зависеть от общего напряжения и обратно пропорционально общему сопротивлению. Увеличение напряжения батареи приведет к увеличению тока в точке P.Уменьшение общего сопротивления приведет к увеличению тока в точке P. Общее сопротивление можно уменьшить, добавив еще один резистор в отдельную ветвь или уменьшив сопротивление любой из ветвей.

51. Три одинаковые лампочки (обозначенные X, Y и Z) подключены к батарее, как показано справа. Какие настройки можно внести в схему ниже, чтобы уменьшить ток в лампочке Z? Перечислите все подходящие варианты.

а. увеличить сопротивление лампы X

г. уменьшить сопротивление лампы X

г. увеличить сопротивление лампы Z

г. уменьшить сопротивление лампы Z

e. увеличить напряжение АКБ

ф. уменьшить напряжение АКБ

г. снять лампу Y

Ответ: CF

Ток в лампе Z зависит от падения напряжения на лампе Z и сопротивления лампы Z.В форме уравнения,

Я Z = V Z / R Z

Увеличение напряжения батареи приведет к увеличению падения напряжения на лампе Z (V Z ) и, таким образом, обеспечит больший ток через лампу. Уменьшение сопротивления лампы Z также приведет к увеличению тока через лампу. Однако изменение положения лампы X или Y не повлияет на соотношение V Z / R Z .



Переход к:

Главная страница сеанса обзора — Список тем

Electric Circuits — Главная || Версия для печати || Вопросы со ссылками

Ответы на вопросы: Все || # 1-7 || # 8-51 || # 52-59 || # 60-72

Вам тоже может понравиться…

Пользователи The Review Session часто ищут учебные ресурсы, которые предоставляют им возможности для практики и обзора, которые включают встроенную обратную связь и инструкции. Если это то, что вы ищете, то вам также может понравиться следующее:
  1. Блокнот калькулятора

    Блокнот калькулятора включает текстовые задачи по физике, организованные по темам. Каждая проблема сопровождается всплывающим ответом и аудиофайлом, в котором подробно объясняется, как подойти к проблеме и решить ее.Это идеальный ресурс для тех, кто хочет улучшить свои навыки решения проблем.

    Посещение: Панель калькулятора На главную | Блокнот для калькулятора — электрические схемы

  2. Minds On Physics App Series

    Minds On Physics the App («MOP the App») представляет собой серию интерактивных модулей вопросов для учащихся, которые серьезно настроены улучшить свое концептуальное понимание физики. Каждый модуль этой серии посвящен отдельной теме и разбит на подтемы.«Опыт MOP» предоставит учащемуся сложные вопросы, отзывы и помощь по конкретным вопросам в контексте игровой среды. Он доступен для телефонов, планшетов, Chromebook и компьютеров Macintosh.

Добавить комментарий

Ваш адрес email не будет опубликован. Обязательные поля помечены *